ielts-material

IELTS Direct Question Essays – Structure, Questions, Samples & Tips

Ann Smith

Updated On Nov 28, 2023

arrow

Share on Whatsapp

Share on Email

Share on Linkedin

IELTS Direct Question Essays – Structure, Questions, Samples & Tips

Limited-Time Offer : Access a FREE 10-Day IELTS Study Plan!

The IELTS Writing Task 2, has an essay type known as the “Direct Question Essay,” which is a crucial component of the IELTS exam. It evaluates your ability to express your ideas, analyze a given topic, and provide a clear, well-structured response.

In this comprehensive guide, we will explore the structure of  IELTS Writing Task 2 : Direct Question Essays, delve into essential pointers, provide ten valuable tips to ace this task and furnish you with five sample questions and answers to boost your preparation.

Structure of a Direct Question Essay

A Direct Question Essay typically follows a specific format:

.

I agree that schools should place a greater emphasis on practical life skills. While traditional subjects are essential, practical skills like cooking, budgeting, and home repair equip students for real-life challenges. For instance, teaching basic culinary skills can promote healthier eating habits, reducing the prevalence of diet-related health issues.

Furthermore, budgeting knowledge empowers individuals to manage their finances effectively, reducing the risk of falling into debt. The ability to handle basic home repairs fosters independence and self-reliance, which are valuable qualities in adulthood.

By incorporating practical skills into the curriculum, schools prepare students for the responsibilities they will face in their daily lives. While academic knowledge is important, it is equally crucial to equip individuals with the tools they need to thrive in the real world. Thus, I firmly believe that schools should allocate more resources to teach practical life skills alongside traditional subjects, ensuring a holistic education that sets students on the path to success.

Banning smoking in public places offers numerous advantages. Firstly, it significantly improves public health by reducing exposure to harmful secondhand smoke. Non-smokers, including children and those with respiratory conditions, benefit from cleaner air, leading to a lower incidence of smoking-related health issues.

Secondly, such bans encourage individuals to quit smoking or reduce their consumption. When smoking is restricted in public spaces, smokers may be more inclined to quit as the inconvenience of finding a designated smoking area becomes apparent. This, in turn, decreases the overall demand for tobacco products and contributes to public health.

However, banning smoking in public places can lead to potential disadvantages. Businesses reliant on tobacco sales may experience a decline in revenue, potentially leading to layoffs or closures. Additionally, some smokers may resist the bans, leading to enforcement challenges and potential conflicts in public spaces.

In conclusion, the advantages of banning smoking in public places, such as improved public health and reduced smoking rates, outweigh the disadvantages. While businesses may face challenges, the long-term benefits to society as a whole are substantial.

Key Pointers for Direct Question Essays

  • Understand the Question:  Read the question carefully to grasp its requirements and focus. Identify keywords that dictate the scope of your response.
  • Plan Your Essay:  Spend a few minutes brainstorming and outlining your essay. Organize your thoughts and main ideas. A well-structured essay flows more naturally.
  • Clarity and Coherence:  Use clear and concise language. Ensure that your essay is easy to follow and logically organized. Employ appropriate transitions between sentences and paragraphs.
  • Stay on Topic:  Avoid straying from the main question. Irrelevant information can reduce the clarity of your essay and affect your score.
  • Word Count:  Adhere to the word count requirement. Going over or under the limit can result in point deductions.
  • Vocabulary and Grammar:  Showcase a wide vocabulary range and accurate grammar. Simple and complex sentences should be used appropriately to enhance the quality of your writing.
  • Examples and Details:  Support your ideas with relevant examples, facts, and details. This strengthens your arguments and demonstrates a deep understanding of the topic.
  • Counter Arguments:  Address counterarguments when relevant. This shows your ability to consider different perspectives and strengthens your position.
  • Time Management:  Allocate time wisely. Ensure you have enough time to review and edit your essay before submission.
  • Revision:  Always review and edit your essay. Look for errors in spelling, grammar, and clarity. A well-edited essay leaves a positive impression.

You can practice more Direct Question Essays  here .

Direct Question Essay: A Skill to Learn!

In conclusion, mastering the art of writing Direct Question Essays is achievable with practice, a clear understanding of the format, and attention to key pointers. By following the structure, incorporating the tips provided, and analyzing the sample questions and answers, you can significantly enhance your performance in the IELTS Writing Task 2. So, start practicing and make your writing shine on test day. Good luck!

Here are the 10 examples for the Direct question essay

Also, check :

  • Tips to Improve IELTS Writing Skills
  • IELTS Writing recent actual test
  • IELTS Band 9 essays
  • Advantage and Disadvantage Essays
  • IELTS Writing Task 1 Connectors

Frequently Asked Questions

How will you write an introduction in the IELTS Direct Question Essay?

What are the common errors the examinees make in the IELTS Direct Question Essay?

How can you score high marks for task achievement in the IELTS Direct Question Essay?

Can I use only one body paragraph in my IELTS Direct Question Essay?

Are IELTS Direct Question Essay and IELTS Double Question Essay the same?

Practice IELTS Writing Task 2 based on Essay types

ielts img

Start Preparing for IELTS: Get Your 10-Day Study Plan Today!

Ann Smith

Explore other Direct Question Essays

Some People Say the Main Way to Be Happy in Life is to Have a Lot of Money – IELTS Writing Task 2

Raajdeep Saha

Is freedom of speech necessary in a free society? – IELTS Writing Task 2

Kasturika Samanta

Many People Like to Wear Fashionable Clothes – IELTS Writing Task 2

Janice Thompson

Consumers Are Faced With Increasing Numbers of Advertisements From Competing Companies – IELTS Writing Task 2

Post your Comments

direct questions essay

Posted on Apr 15, 2022

Thank you very much for sharing this useful post.I’m sure this educational programs & sectors will be helpful for those who want to improve their skills in learning better English. So I expect you going on this.

Recent Articles

Best Way to Produce a Happier Society – IELTS Writing Task 2

Akanksha Tripathi

IELTS Writing Task 2 Sample Essays

IELTSMaterial Master Program

1:1 Live Training with Band 9 Teachers

4.9 ( 3452 Reviews )

Our Offices

Gurgaon city scape, gurgaon bptp.

Step 1 of 3

Great going .

Get a free session from trainer

Have you taken test before?

Please select any option

Email test -->

Please enter Email ID

Mobile Band 9 trainer -->

Please enter phone number

Application

Please select any one

Already Registered?

Select a date

Please select a date

Select a time (IST Time Zone)

Please select a time

Mark Your Calendar: Free Session with Expert on

Which exam are you preparing?

Great Going!

  • Phone: +91 8466016171
  • Whatsapp: +91 8208375580
  • Email: contact@leapscholar.com

Direct Question Essay in IELTS: Structure and Examples

  • Updated On June 14, 2024
  • Published In IELTS Preparation 💻

The International English Language Testing System (IELTS) is one of the globally-recognised English proficiency exams. It is designed to assess the language skills of non-native English speakers.  With over 3 million tests taken annually, it is accepted by more than 12,000 organisations worldwide, including universities, employers, and immigration authorities. In the IELTS Writing section, two tasks test different aspects of writing ability- Task 1 and Task 2- which we will explain in detail later .  The Direct Question Essay in IELTS is a crucial component of the IELTS Writing Task 2 , often challenging you to address specific inquiries directly. Understanding this essay type is essential for achieving a high band score, as it assesses your ability to construct well-organised arguments and provide insightful responses to given prompts. 

Table of Contents

This blog delves into the definition, structure, and strategies for mastering Direct Question Essays structure, offering valuable insights and examples to guide you in your IELTS preparation.

Before getting started, let’s take you through the key details.

Opinion
Discussion
Advantage/Disadvantage
Problem/Solution
Direct Question
Academic Task 1: Visual data description
Academic Task 2: Argumentative essay
General Task 1: Letter writing
General Task 2: Direct Question, Opinion, Argument, or Problem-Solution essay
Requires direct responses to specific questions in IELTS Writing Task 2.

 





TASK 1 and TASK 2 in IELTS 

The IELTS Writing section is a critical component of the exam, challenging candidates to demonstrate their proficiency in different types of writing. Each task has its unique requirements and focuses on distinct skills, making it essential to understand the nuances of both tasks for effective preparation. The tasks also differ in the Academic and General Training version of the IELTS test. Let’s walk you through the differences between the two tasks.

Crack IELTS in 4 Weeks! IELTS 8 Band Guarantee

study in canada

Ace IELTS with Band Guarantee. Get access to Live Classes & 100+ Mock Test. Join Free IELTS Masterclass Now!

LA Banner Web

IELTS Academic Writing Tasks

Another important factor to keep in mind is that Task 1 and Task 2 under Academic writing also differ. Now, let’s show you the differences for you to ace your exam!

DescriptionDescribe, summarise, or explain visual information (graph, table, chart, diagram)Write an essay in response to a point of view, argument, or problem
Word Count
Time

Objectives and Examples of Task 1 and 2 Academic IELTS Task 1 (Academic) aims to assess your ability to identify key features, trends, and significant data in visual representations like graphs, charts, and tables. You are expected to describe the information accurately, highlight important trends, and compare and contrast data points where relevant.

For example, if you are given a line graph showing population growth in different countries over time, your task would be to describe the overall population growth trend, identify any significant fluctuations or patterns, and compare the growth rates between countries if applicable.

On the other hand, IELTS Task 2 (Academic) aims to evaluate your ability to present a clear and relevant argument, support it with evidence and examples, and demonstrate coherent and cohesive writing skills. You will be given a topic or statement and asked to discuss it from multiple perspectives, presenting both sides of the argument and providing your opinion or stance.

For instance, a Task 2 prompt could ask you to discuss both sides of an argument about whether governments should spend more on education. Here, you would need to present arguments supporting increased government spending on education and arguments against it, backed by evidence and examples. Additionally, you would offer your opinion while ensuring your essay is well-structured, coherent, and logically developed.

IELTS General Training Writing Tasks

Did you know that Task 1 and task 2 under General Training differ on the basis of what is supposed to be done, the word count and also the time duration? Let’s take you through the differences.

DescriptionWrite a letter in response to a given situation (formal, semi-formal, or informal)Write an essay in response to a point of view, argument, or problem
Word Count
Time

Objectives and Examples of Task 1 and 2 General Training

The IELTS General Training Writing Task 1 aims to communicate practical information, request information, or explain a situation through a letter. 

For example, you might be asked to write a letter to a friend inviting them to visit and explain the details of their stay. This involves giving clear and concise information about the visit, such as dates, accommodation, activities planned, and any other relevant details to ensure your friend has a good understanding of what to expect. It’s essential to use a polite and friendly tone and structure your letter in a coherent and organised manner.

In IELTS General Training Writing Task 2, the objective is to present and justify an opinion, compare and contrast evidence, and demonstrate coherent and cohesive writing.  

For instance, you might be asked to discuss the benefits and drawbacks of living in a big city. This task requires you to consider both sides of the argument, provide examples and evidence to support your points and organise your essay logically and clearly. You should aim to write in a balanced and objective way, showing your ability to construct a well-reasoned argument and to link your ideas effectively.

Crack IELTS Exam in first attempt

Direct Question Essay in IELTS: Structure and Examples

Attend Leap’s free masterclass to get tips, tricks and advance strategies to crack IELTS exam in first attempt

Direct Question Essay in IELTS: Structure and Examples

Types of IELTS Essays for Writing Task 2 Topics

You may encounter different essay prompts in the IELTS Writing Task 2. Understanding these types is crucial for crafting well-structured responses. Most IELTS test-takers also believe that familiarising themselves with essay types significantly improves their writing scores. Understanding these types helps structure responses effectively. Here are the common types:

  • Description : These essays ask you to agree or disagree with a statement or to what extent you agree/disagree.
  • Example : “To what extent do you agree or disagree with the statement: ‘Technology has made our lives more complex.'”
  • Description : These require you to discuss two opposing views and give your own opinion.
  • Example : “Some people believe that higher education should be free for everyone, while others think students should pay for their studies. Discuss both views and give your opinion.”
  • Description : These ask you to discuss the advantages and disadvantages of a particular issue.
  • Example : “What are the advantages and disadvantages of using public transportation?”
  • Description : These require you to identify problems related to a particular issue and propose solutions.
  • Example : “What are the major environmental issues facing the world today, and what measures can be taken to address them?”
  • Description : These essays ask a direct question and expect a specific answer.
  • Example: “The use of social media has grown significantly in recent years. What impact does this have on relationships and communication?”

Now, let’s walk you step by step through the important aspects of the Direct Question Essay in IELTS.

What is Direct Question Essay in IELTS?

In the context of IELTS, a Direct Question Essay requires you to respond to one or more questions in the prompt. 

Unlike other essay types, such as opinion or discussion essays, the Direct Question Essay in IELTS demands a focused response that comprehensively addresses each part of the question. This essay type tests your ability to provide clear, coherent, and relevant answers, showcasing your language proficiency and critical thinking skills.

Importance of Direct Question Essay in IELTS Writing Task 2

Direct Question Essay in IELTS plays a significant role in IELTS Writing Task 2, often appearing in exams. 

It evaluates your ability to understand and interpret questions accurately, structure your responses logically, and support your arguments with relevant examples. 

Achieving proficiency in this essay type can significantly impact your overall writing score, making it a vital area of preparation for IELTS candidates.

Excel in IELTS with Expert Guidance! Contact Leap Scholar to learn from India’s best-rated IELTS trainers , with an average student rating of 4.5/5.

Sample Questions and Approaches

Understanding how to approach different Direct Question Essay topics can significantly enhance your preparation. This section provides sample questions and outlines model answer approaches to guide you.

Sample Question and Answer on the Importance of Practical Skills in Education

Sample Question : Some people believe that practical skills are more important than theoretical knowledge in education. To what extent do you agree or disagree? Provide examples to support your opinion.

Model Answer Outline :

Direct Question Essay in IELTS: Structure and Examples

  • Introduction: Paraphrase the question, provide background information on practical skills vs. theoretical knowledge, and state the thesis.
  • Body Paragraph 1: Discuss the importance of practical skills, providing examples from vocational training.
  • Body Paragraph 2: Highlight the role of theoretical knowledge, giving examples from academic disciplines.
  • Conclusion: Summarise the main points and reiterate the balanced view on the importance of both practical skills and theoretical knowledge.

SAMPLE ANSWER Practical Skills vs. Theoretical Knowledge in Education

The debate over whether practical skills or theoretical knowledge are more important in education has been a longstanding one. Some people believe that practical skills are more beneficial, while others argue that theoretical knowledge forms the backbone of education. In this essay, I will explore both perspectives and argue that a balanced approach, incorporating both practical skills and theoretical knowledge, is essential for a comprehensive education.

Importance of Practical Skills

From my experience, practical skills are crucial because they prepare individuals for real-world applications. Vocational training, for example, focuses on hands-on experience, teaching students how to apply their knowledge in practical settings. In fields such as plumbing, carpentry, and the culinary arts, practical skills are paramount. When I studied culinary arts, I found that spending time in the kitchen, learning techniques and recipes firsthand, was invaluable. This hands-on experience equipped me with the skills necessary to excel in my chosen profession.

Role of Theoretical Knowledge

On the other hand, theoretical knowledge provides the foundational understanding necessary for practical application. Academic disciplines such as medicine, engineering, and the sciences rely heavily on theoretical frameworks. When I was studying medicine, I realised that grasping complex biological and chemical principles was essential before I could effectively diagnose and treat patients. Similarly, an engineering student needs to understand physics and mathematics to design and build safe structures. Without a strong theoretical foundation, practical skills alone may be insufficient and potentially dangerous.

In conclusion, I believe that both practical skills and theoretical knowledge play vital roles in education. Practical skills provide the hands-on experience needed for real-world applications, while theoretical knowledge offers the essential foundation for understanding and innovation. A balanced approach, integrating both elements, ensures that students are well-equipped to succeed in their careers and contribute meaningfully to society.

Sample Question on Discussing the Role of the Internet and Information Reliability with Examples

Sample Question : The internet provides a vast amount of information, but the reliability of this information is often questioned. Discuss the impact of the internet on information reliability, providing examples.

  • Introduction: Paraphrase the question, discuss the prevalence of internet information, and state the thesis.
  • Body Paragraph 1: Examine the benefits of internet information, providing examples of accessible educational resources.
  • Body Paragraph 2: Discuss the challenges of information reliability, giving examples of misinformation and fake news.
  • Conclusion: Summarise main points and emphasise the need for critical evaluation of online information.

Sample Question on Addressing Happiness, Success Measurements, and Their Nuances

Sample Question : Some people think that happiness is the key to success, while others believe that success leads to happiness. Discuss both views and give your opinion.

  • Introduction: Paraphrase the question, provide context on happiness and success, and state the thesis.
  • Body Paragraph 1: Discuss the view that happiness leads to success, giving examples from personal well-being and productivity.
  • Body Paragraph 2: Explore the perspective that success brings happiness, providing examples from career achievements and financial stability.
  • Conclusion: Summarise the main points and present a balanced opinion on the interrelationship between happiness and success.

SAMPLE ANSWER

The Interrelationship Between Happiness and Success

The debate over whether happiness is the key to success or success leads to happiness has intrigued many. Some argue that being happy naturally leads to success, while others contend that achieving success brings about happiness. In this essay, I will discuss both viewpoints and provide my opinion on the interrelationship between happiness and success.

Happiness Leads to Success

One perspective is that happiness fosters success. When individuals are happy, they tend to have better mental health and higher energy levels, which can lead to increased productivity and creativity. For instance, I have noticed that when I am in a good mood, I am more focused and motivated to work hard, which often results in better outcomes. Furthermore, happy people are generally more resilient and capable of handling stress, which can contribute to achieving long-term goals. This suggests that a positive state of mind can significantly enhance one’s chances of success.

Success Brings Happiness

On the other hand, some believe that success brings happiness. Achieving career milestones or financial stability can lead to a sense of accomplishment and security, which in turn fosters happiness. For example, when I achieved a major career goal, the recognition and rewards that followed significantly boosted my overall happiness. Similarly, financial success can alleviate stress and provide opportunities for enjoying life, such as travelling or pursuing hobbies. This viewpoint implies that success can create the conditions necessary for a happy and fulfilling life.

In conclusion, both happiness and success are closely intertwined and can influence each other. Happiness can enhance productivity and resilience, leading to success, while success can provide the security and satisfaction that contribute to happiness. I believe that fostering happiness and striving for success should go hand in hand, as both elements are essential for a well-rounded and fulfilling life.

Sample Question on Cultural Adaptation During Travel

Travelling to a new country requires adapting to its culture. Discuss the importance of cultural adaptation during travel, providing examples.

  • Introduction: Paraphrase the question, discuss the necessity of cultural adaptation, and state the thesis.
  • Body Paragraph 1: Analyse the benefits of cultural adaptation, giving examples from language learning and social integration.
  • Body Paragraph 2: Examine the challenges of cultural adaptation, providing examples from cultural misunderstandings and adjustment difficulties.
  • Conclusion: Summarise the main points and emphasise the significance of embracing cultural diversity during travel.

Achieve Your Dream IELTS Score! Get in touch with Leap Scholar and learn from India’s top IELTS trainers.

Sample Question on Exploring the Significance of Art and Music in the Technological Era

In today’s technological era, what is the significance of art and music? Provide examples to support your answer.

  • Introduction: Paraphrase the question, discuss the technological era’s impact on art and music, and state the thesis.
  • Body Paragraph 1: Highlight the enduring relevance of art, giving examples from contemporary visual arts and digital media.
  • Body Paragraph 2: Discuss the importance of music, providing examples from the music industry’s evolution and digital platforms.
  • Conclusion: Summarise the main points and reaffirm the vital role of art and music in the technological age.

Understanding the Essay Structure

Did you know IELTS is available in over 4,000 test locations in 140 countries? Mastering the essay structure is crucial to writing effective direct-question essays in IELTS. This section breaks down the key components and provides tips for crafting each part effectively.

Introduction: Paraphrase the Question and Provide Background Information

Your introduction should begin by paraphrasing the question, demonstrating an understanding of the prompt. Some background information helps set the context, making it easier for readers to follow your subsequent arguments. This section should also include a thesis statement outlining the main points discussed in the essay.

Body Paragraph 1: Discuss the First Aspect of the Question with Examples

The first body paragraph should address the initial aspect of the question, offering a detailed discussion supported by relevant examples. The initial aspect refers to the first part of the question or the first issue presented. For instance, in the question, “The use of social media has grown significantly in recent years. What impact does this have on relationships and communication?” the initial aspect would be “the positive impact of social media on relationships and communication.” 

This paragraph should present a clear argument, ensuring each point is well-explained and logically connected to your essay’s overall thesis.

Body Paragraph 2: Examine the Second Aspect of the Question, Providing Explanations and Examples

The second body paragraph should focus on the other aspect of the question , providing thorough explanations and examples. The second aspect refers to the second part of the question or the secondary approach. For instance, in the question, “The use of social media has grown significantly in recent years. What impact does this have on relationships and communication?” The next aspect would be “the negative impact of social media on relationships and communication.” 

This section should build upon the points in the first body paragraph, offering additional insights and reinforcing your essay’s main arguments.

Conclusion: Summarise Main Points and Reiterate Your Opinion

Your conclusion should briefly summarise the main points discussed in the essay, reiterating the thesis statement. It should provide a clear and concise wrap-up, leaving a lasting impression on the reader. This section should not introduce new information but rather reinforce the key arguments presented earlier.

Also Read: 60+ IELTS Essay Topics 2024: IELTS Writing Task 2 Samples & Tips

Strategies for Writing Direct Question Essay in IELTS

Are you aware that around 80% of IELTS test-takers take the exam for academic purposes, such as university admission, while 20% take it for immigration and employment purposes? 

To write effective Direct Question Essays in IELTS, you should employ various strategies that ensure your responses are clear, coherent, and well-supported. This section outlines essential techniques for mastering this essay type.

Time Management: Dividing Time for Planning, Writing, and Reviewing

Effective time management is crucial for writing a well-structured essay. Allocate sufficient planning, writing, and reviewing time to ensure a comprehensive and polished response. Here is a table showcasing approximately how much time you should spend on each stage.

Planning
Writing
Reviewing

Answer Planning: Structuring Answers to Each Question for Coherence

Planning your answer involves outlining the main points and examples for each question. This helps maintain coherence and ensures that all aspects and possibilities of the question are addressed. Creating a clear outline before writing can enhance your essay’s structure and flow.

Importance of Paraphrasing the Question in the Introduction

Paraphrasing the question in the introduction demonstrates comprehension and sets the tone for the essay. It helps avoid repetition and shows the examiner that you can interpret and articulate the question in your own words.

Developing Arguments: Supporting Main Points with Examples and Explanations

Strong arguments are essential for a high-scoring essay. Each main point should be supported with relevant examples and detailed explanations. This strengthens the argument and showcases your ability to provide evidence and logical reasoning.

Example 1: Positive Impact of Social Media on Relationships  

Main Point: Social media helps maintain relationships over long distances. 

Explanation: With platforms like Facebook and Instagram, people can stay updated on their friends’ and family members’ lives, even if they are geographically separated. 

Example: For instance, a person living in the United States can easily keep in touch with a friend in Australia by sharing photos, sending messages, and having video calls. This continuous interaction helps maintain a strong bond despite the physical distance.

Example 2: Negative Impact of Social Media on Relationships  

Main Point: Social media can lead to superficial relationships. 

Explanation: Online interactions often lack the depth and emotional connection of face-to-face conversations, leading to more shallow relationships. 

Example: For example, a study by the Pew Research Center found that while people may have hundreds of friends on social media, they often feel lonelier and have fewer close friends with whom they can share personal issues. This indicates that social media can contribute to a sense of isolation despite an increase in the number of online connections.

Get the Best Live Learning Experience! Sign up with Leap Scholar for live IELTS online classes and 6-month access to class recordings.

Ensuring Coherence and Clarity in the Essay

Coherence and clarity are vital for effective communication. Ensure each paragraph transitions smoothly to the next, maintaining a logical flow. Use cohesive devices such as linking words and phrases to enhance readability.

Example: Paragraph Transition: “Despite the many benefits of social media, there are also significant drawbacks that cannot be ignored.” 

Using Linking Words and Phrases: “Firstly, social media can create superficial relationships, as interactions online often lack the emotional depth of face-to-face conversations. Additionally, it can lead to misunderstandings due to the absence of non-verbal cues. Moreover, excessive use of social media can result in addiction, thereby reducing the time available for meaningful, in-person interactions.”

Before: “Social media helps maintain relationships. People can stay updated on their friends’ lives. It can lead to superficial relationships. Online interactions lack depth.”

After (with coherence and clarity): “Social media helps maintain relationships by allowing individuals to stay updated on their friends’ lives through platforms like Facebook and Instagram. However, it can also lead to superficial relationships, as online interactions often lack the depth and emotional connection of face-to-face conversations. This duality highlights the complex impact of social media on modern relationships.”

Developing strong arguments with examples and ensuring coherence and clarity will make your essay well-structured and persuasive.

Vocabulary and Grammar: Utilising a Broad Lexical Range and Accurate Grammar

A wide range of vocabulary and accurate grammar are vital components of a high-scoring essay. Use varied and sophisticated language to convey ideas clearly and effectively. Attention to grammar and punctuation to avoid errors that can detract from the essay’s overall quality.

Lexical range refers to the variety and sophistication of the words and phrases you use in your writing. A broad lexical range demonstrates your ability to use different words to express similar ideas, avoiding repetition and making your writing more engaging and precise. It involves using synonyms, advanced vocabulary, varied sentence structures, idioms and phrases.

Join the PAN India IELTS Community! Reach out to Leap Scholar and start your journey towards a high IELTS score.

Check Out: IELTS Test Online: Free Sample Papers & Study Guide for 2024

Key Tips for Direct Question Essay Success

Mastering Direct Question Essays in IELTS requires careful attention to detail and strategic planning. This section offers vital tips for improving your essay writing skills and achieving higher scores. Let’s explain you the points with relevant examples on the topic: Impact of Social Media on Relationships and Communication.

Direct Question Essay in IELTS: Structure and Examples

Understanding and Analysing the Questions Carefully

Carefully analyse questions to provide focused responses. Address all parts of the question.

  • For example, if asked about the impact of social media on communication, cover both positive and negative aspects.

Staying on Topic and Avoiding Irrelevant Information

Staying on topic is essential for maintaining coherence and clarity. Avoid including irrelevant information that does not directly relate to the question or support the main arguments. 

  • For instance: When discussing the benefits of social media, avoid tangents about unrelated technologies.

The Role of Examples in Strengthening Arguments

Examples strengthen arguments by providing evidence. Use specific, relevant examples to illustrate your points. 

  • For example, mention how social media enabled remote communication during the COVID-19 pandemic.

The Need for Counterarguments to Showcase Critical Thinking

Counterarguments showcase critical thinking and provide a balanced perspective. Address and refute opposing viewpoints with logical reasoning. 

  • For example, while social media enhances connectivity, it can lead to addiction—suggest moderation to mitigate this.

Grammar and Vocabulary: Making the Essay Stand Out

Use a broad vocabulary and accurate grammar to make your essay stand out. Employ varied, sophisticated language and ensure correct grammar and punctuation. 

  • For instance, instead of “important,” use “paramount” or “crucial” to add depth.

By following these guidelines and strategies, you can master the Direct Question Essay in IELTS, significantly improving your writing score and overall exam performance.

Also Read: IELTS Writing Samples 2024: Task 1 & Task 2 (Sample Questions & Tips)

In conclusion, mastering the Direct Question Essay in IELTS Writing Task 2 requires a strategic approach and thorough preparation. Focus on structuring your essay clearly, beginning with an introduction that paraphrases the question and sets the context. 

Develop detailed body paragraphs that address each aspect of the question with relevant examples, and conclude by summarising your main points.

Effective time management is key—plan, write, and review within the allotted time. Outlining your main points and examples will ensure coherence. You can also enhance your essay with varied vocabulary and accurate grammar. Incorporating personal examples can add authenticity, while counter arguments demonstrate critical thinking.

By following these strategies and practising consistently, you can confidently tackle Direct Question Essays and achieve a higher band score in the IELTS exam. Each essay is an opportunity to showcase your language proficiency and communication skills, so refine your approach and deliver your best performance.

Ready for IELTS Success? Connect with Leap Scholar today and join 90% of our students who score 7+ bands!

Frequently Asked Questions

Q. what is the ielts exam.

A. The International English Language Testing System (IELTS) is a standardised test designed to measure the English language proficiency of non-native English speakers. The British Council, IDP: IELTS Australia, and Cambridge Assessment English jointly manage it. IELTS is widely recognised by educational institutions, employers, and immigration authorities in over 140 countries.

Q. What are the different versions of the IELTS test?

A. There are two versions of the IELTS test: Academic : This version is for those applying for higher education or professional registration. General Training : This version is for those migrating to an English-speaking country or seeking secondary education, work experience, or training programmes.

Q. How is the IELTS test structured?

A. The IELTS test comprises four sections: Listening : 30 minutes Reading : 60 minutes Writing : 60 minutes Speaking : 11-14 minutes The total test duration is 2 hours and 45 minutes.

Q. How is the IELTS speaking test conducted?

A. The IELTS speaking test is a face-to-face interview with a certified examiner. It lasts 11-14 minutes and is divided into three parts: Part 1 : Introduction and interview (4-5 minutes) Part 2 : Long turn (3-4 minutes, including preparation time) Part 3 : Discussion (4-5 minutes)

Q. How long are IELTS scores valid?

A. IELTS scores are valid for two years from the test date. After this period, test takers may need to retake the exam to provide up-to-date evidence of their English language proficiency.

Q. How much does the IELTS test cost?

A. The cost of the IELTS test varies by country and test centre. On average, the test fee ranges around INR 17K . It is recommended to check with your local test centre for the exact fee in your region.

Q. How soon will I receive my IELTS test results?

A. IELTS test results are typically available 13 days after the test date for paper-based tests. For computer-delivered tests, results are usually available within 5-7 days. You can access your results online, and a Test Report Form (TRF) will be sent to you by post.

Q. What is a Direct Question Essay in the IELTS Writing Task 2?

A. In the IELTS Writing Task 2, a Direct Question Essay requires you to respond to one or more questions directly related to a given topic. You need to address each question in a structured manner, typically including an introduction, body paragraphs for each question, and a conclusion. This essay type tests your ability to answer specific questions clearly and coherently.

Q. What is Direct Question Essay structure in IELTS Writing Task 2?

A. A well-structured Direct Question Essay template should include: Introduction : Paraphrase the question and provide background information. Body Paragraph 1 : Discuss the first aspect of the question with examples. Body Paragraph 2 : Examine the second aspect of the question, providing explanations and examples. Conclusion : Summarise the main points and reiterate your opinion.

Q. What are some strategies for writing a successful Direct Question Essay?

A. To write a successful Direct Question Essay: Time Management : Allocate time for planning, writing, and reviewing. Answer Planning : Structure your answers to ensure coherence. Paraphrasing : Paraphrase the question in the introduction to show understanding. Arguments and Examples : Develop arguments supported by relevant examples. Coherence and Clarity : Ensure your essay is coherent and easy to follow. Vocabulary and Grammar : Use a broad range of vocabulary and accurate grammar.

Q. Can you provide a sample question for a Direct Question Essay and an outline for the answer?

A. Sure! Here is a sample question: – Question : “Some people think that practical skills are more important than theoretical knowledge. To what extent do you agree or disagree?” – Outline : 1. Introduction : Paraphrase the question and state your opinion. 2. Body Paragraph 1 : Discuss the importance of practical skills with examples. 3. Body Paragraph 2 : Examine the role of theoretical knowledge, providing explanations and examples. 4. Conclusion : Summarise the main points and restate your opinion. 5. Stay on Topic : Ensure all parts of your essay address the question directly.

Q. Can I use personal examples in a Direct Question Essay?

A.  Yes, you can use personal examples in a Direct Question Essay. Personal examples can make your essay more relatable and authentic. However, ensure that the examples are relevant to the question and support your arguments effectively. Personal examples should be clear, concise, and directly related to the topic.

Know more about IELTS

Get free speaking practice samples.

  • Describe a Skill That you can teach other people
  • Describe a Place you Visited Where the Air was Polluted
  • Describe a Famous Person that you are Interested in
  • Describe a Course That You Want to Learn
  • Describe a Person who Solved a Problem in Smart Way
  • Describe a Prize That You Received
  • Describe a Volunteering Experience You Have Had
  • Describe a Piece of Good News
  • Describe Something you Taught to Your Friend
  • Talk About an Interesting Old Person you Met Recently
  • Describe a Dinner you Really Enjoyed
  • Describe a Story or Novel That Was Interesting to You
  • Describe a Time When you Shared Something with Others
  • Describe a Toy you Liked in Childhood
  • Describe an Interesting Neighbour
  • Describe a Competition You Would Like to Take Part In
  • Describe an interesting activity that you remember enjoying most in your Primary School
  • Describe Your Childhood Friend
  • Describe an Object You Find Particularly Beautiful
  • Describe a Place Where you are Able to Relax
  • Describe a person you know who likes to talk a lot
  • Describe a place where you would like to go to relax
  • Describe a period when you were busy
  • Describe a long-term goal you would like to achieve
  • Describe a situation when you helped someone
  • Describe a job you would not like to do in the future
  • Describe a time when you used a map
  • Describe a skill that you learned in your childhood
  • Describe an unusual holiday you had
  • Describe an exciting activity that you experienced with someone else
  • Describe a person who is good at making people feel welcome in his/her home
  • Describe A Time You Successfully Did Something Difficult
  • Describe Something in Your Country That You Are Interested In
  • Describe a Beautiful City
  • Describe something you do regularly that helps you work and study better
  • Describe a photo that you took and are proud of
  • Describe a party that you enjoyed
  • Describe a useful object in your home that you can’t live without
  • Describe a good advertisement that you think is useful
  • Describe an occasion when someone or something made noise
  • Describe a sportsperson that you admire
  • Describe something that you picked up that was thrown by someone else
  • Describe a time that something changed your life in good ways
  • Describe a successful person who you once studied or worked with

Get Free Reading Practice Samples

  • William henry perkin reading answers
  • Why zoos are good reading answers
  • Bioluminescence reading answers
  • Classifying societies reading answers
  • Artificial artists reading answers
  • Jargon reading answers
  • The Origins of Laughter Reading Answers
  • The Innovation of Grocery Stores Reading Answers
  • All About the Otter Reading Answers
  • The Triune Brain Reading Answers
  • Saving the Soil Reading Answers
  • Trans Fatty Acids Reading Answers
  • A Disaster of Titanic Proportions Reading Answers
  • Why Companies Should Welcome Disorder Reading Answers
  • Book Review Reading Answers
  • Tea Times Reading Answers
  • Why are Finland's Schools Successful Reading Answers
  • Intelligence and Giftedness Reading Answers
  • Animal Minds Parrot Alex Reading Answers
  • Crop Growing Skyscrapers Reading Answers
  • Secrets of the Swarm Reading Answers
  • Walking with Dinosaurs Reading Answers
  • The Development of Travel Under the Ocean Reading Answers
  • What's so Funny Reading Answers
  • The Culture of Chimpanzee Reading Answers
  • Clutter Bugs Beware Reading Answer
  • Stepwells Reading Answers
  • Glaciers Reading Answers

Get Free Writing Practice Samples

  • Advantages and Disadvantages Essay with Sample Answers
  • Agree and Disagree Essay with Sample Answers
  • Problem Solution Essay Topic with Sample Answers
  • Every year several languages die out
  • Positive or Negative Development Essay with Sample Answers
  • Honesty is the best policy essay
  • Online shopping essay
  • Environment essay topics
  • Prevention is better than cure essay

Get Free Listening Practice Samples

  • Family excursions listening answers
  • Public library listening answers
  • Hiring a public room listening answers
  • Notes on social programme listening answers
  • Accommodation request form listening answers
  • Transport survey listening answers
  • Mic house agency repairs listening answers
  • Holiday rentals listening answer
  • Job enquiry listening answers
  • Homestay application listening answers
  • Library information listening answers
  • Free activities in the burnham area listening answers

IELTS Important Information

  • IELTS Exam Date
  • IELTS Exam Fee
  • IELTS Modules
  • IELTS Speaking Practice Test
  • IELTS Writing Practice Test
  • IELTS Reading Practice Test
  • IELTS Listening Practice Test
  • IELTS Test Centres
  • IELTS Results
  • Types of IELTS
  • IELTS Pattern
  • IELTS Exam Eligibilty
  • IELTS Slot Booking
  • IELTS Band Score
  • IELTS Registration
  • IELTS Books
  • IELTS Preparation
  • IELTS Practice Test
  • IELTS Speaking Cue Card
  • IELTS Speaking Part 1
  • IELTS Writing Task 1
  • IELTS Writing Task 2
  • Task 1 Pie Chart
  • Task 1 Table Chart
  • Task 1 Bar Graph
  • Task 1 Line Graph
  • Task 1 Diagram
  • IELTS Computer Based Test
  • IELTS Paper Based Test
  • IELTS One Skill Retake
  • IELTS for UKVI
  • IELTS Vocabulary

IELTS Test Centre and Dates in India

  • IELTS Test Centre and Dates in Hyderabad
  • IELTS Test Centre and Dates in Bangalore
  • IELTS Test Centre and Dates in Chennai
  • IELTS Test Centre and Dates in Amritsar
  • IELTS Centre and Dates in Ludhiana
  • IELTS Test Centre and Dates in Mumbai
  • IELTS Test Centres and Dates in Ahmedabad
  • IELTS Centre and Dates in Delhi
  • IELTS Test Centres and Dates in Chandigarh
  • IELTS Center and Dates in Pune

Avatar photo

Kanika Singh

Kanika took a detour from academics for the corporate world. Now she works as a senior editor specialising in content creation, marketing, and strategy.

Related Posts

direct questions essay

IELTS to CLB Calculator: How to Calculate

Dictation Words for IELTS

Dictation Words for IELTS: Practice and Preparation

common words used in the daily life

A Guide on Most Common English Words Used in Daily Life

Trending now.

early childhood

IELTS Charlie

Your Guide to IELTS Band 7

IELTS Direct Question Essay: tips, common mistakes, questions & essays

In this lesson we are going to look at how to answer an IELTS Direct Question essay.

You will learn about this  IELTS Writing Task 2  essay, using  authentic IELTS essay questions , plus the most common mistakes. And I will finish with an  IELTS model essay  written by me in response to a  sample IELTS essay question . So let’s get started!

What Is Your Task?

In this IELTS question type, you are usually asked 2 questions. Your task is to simply answer these questions!

Often, one or both of the 2 questions come from one of the other 4 question types, so you might be asked to say whether you agree or disagree, or to discuss the disadvantages of something, or to suggest some solutions.

So it’s really important to read the question carefully!

Here is an example direct questions essay task:

Some people spend most of their lives living close to where they were born. 

What might be the reasons for this? 

What are the advantages and disadvantages? 

Cambridge IELTS 16 General Training Test 3

As you can see, this task has 2 questions. The 2nd question is about advantages and disadvantages, so the task takes the 2nd question from one of the other essay question types.

How To Plan An IELTS Direct Question Essay

If you are aiming for a high band score (band 7 and above) it is absolutely vital that you plan your essay. A good plan will help you to see if you have answered the question, developed your ideas and organised them BEFORE you start writing.

We’re going to plan an essay using my  4 Step Planning Process .

4 Step Planning Process

Step 1: Understand The Task

First, you need to make sure you understand exactly what you need to write about. So you need to read the question carefully, not quickly!

Think about these three questions:

What is the topic about?

What is the topic NOT about?

How should you respond to the topic?

Let’s go back to this essay question, and answer those 3 questions:

  • The topic is about people who live near to the village / town of their birth for most of their lives.
  • (The word “birthplace” implies the village /  town of their birth, NOT the country of their birth.)
  • It’s not about people who live ALL their lives near their birthplace.
  • Because “birthplace” implies a village / town / city, it’s not about people who rarely go outside their country.
  • The two questions, “what might be the reasons for this?” and “what are the advantages and disadvantages” tell you how to respond to the topic. So make sure you answer these questions in your plan.
  • Many students might forget to answer the first question, and just write about the advantages and disadvantages. This will limit your band score for Task Response to Band 5.

Step 2: Decide Your Position

Next, you need to decide your position. In other words, you need to decide what you think.

In a direct questions essay, your position is your answers to the two questions .

So in our example above, your position is your answer to the 2 questions:

  • what might be the reasons for people spending most of their lives near their birthplace?
  • what are the advantages and disadvantages of spending most of your life near your birthplace?

Step 3: Extend Your Ideas

When you decided your position, you may have started thinking about the reasons for your position, the reasons for your answer. In other words, WHY are you taking this view?

Giving reasons for your view is essential in an IELTS essay. In fact, all IELTS questions tell you to “give reasons for your answer”. So in Step 3, you need to think about your reasons a little more.

However, just presenting your  reasons is not enough. You need to develop them.

The two best ways of developing your ideas is by:

  • giving explanations of what you mean
  • giving specific examples which illustrate what you mean

Together, these add more detail to your answer.

You MUST do this to get Band 7. If you fail to develop your ideas in detail, your band score for Task Response may be limited to Band 6.

Read more about how to develop your ideas in an IELTS essay.

Step 4: Structure Your Essay

The final step in the planning process is to structure your essay. This simply means deciding which main ideas to put in which paragraphs.

I would recommend a simple structure like this:

  • Paragraph 1: introduce the essay
  • Paragraph 2: discuss your answer to the 1st question
  • Paragraph 3: discuss your answer to the 2nd question
  • Paragraph 4: summarise your ideas.

ielts-direct-question-essay

How To Write Your IELTS Direct Question Essay

Let’s go through how to write the different parts of the essay.

How To Write The Introduction

In the introduction to an IELTS Direct Questions essay, you need to do two things:

  • briefly introduce the topic of the essay
  • briefly say what you are going to write about

Introduce The Topic

You should begin with a background sentence which introduces your reader to the topic of the essay. The best way to do this is to paraphrase the topic statement.

How To Paraphrase

Think about the meaning of the topic statement, and briefly rewrite it using your own words. Try not to use the same grammatical structures as in the essay question, and try to move language around. In other words, be flexible. This is important if you are aiming for a Band 7 or higher.

In the example essay question above, the topic statement said:

“Some people spend most of their lives living close to where they were born.”

Here is one way of paraphrasing this:

“Despite opportunities to travel widely in the modern world, many people still live most of their lives not far from their birthplace.”

This sentence has a similar meaning as the original sentence, but uses different vocabulary and different grammatical structures.

Say What You Are Going To Write About

In an IELTS Direct Questions essays, it’s a good idea to briefly say what you are going to write about – in other words, say that you are going to answer the two questions.

In our example essay above, we need to answer two questions:

  • What might be the reasons for this?
  • What are the advantages and disadvantages?

So I could write:

“This essay will consider the reasons for this tendency, along with the possible benefits and drawbacks.”

How To Write The Body Paragraphs

In an IELTS Direct Questions essay, you simply need to answer the questions in the body paragraphs.

Write the answer to each question in a separate paragraph.

Direct questions essays can vary quite a lot, so you need to be flexible in your paragraphing.

In our example essay, the two questions are:

So, the first body paragraphs could contain:

  • A reason why people live most of their lives near their birthplace
  • A more detailed explanation of this reason
  • An example which illustrates this reason
  • You can also include a 2nd reason in the same paragraph.

The second body paragraph could contain:

  • One advantage of living most of their life near their birthplace
  • A more detailed explanation of this advantage
  • An example which illustrates this advantage

You could include a disadvantage in the same paragraph, but I would personally write it in a separate paragraph:

  • One disadvantage of living most of their life near their birthplace
  • A more detailed explanation of this disadvantage
  • An example which illustrates this disadvantage

(You probably only have enough time to write one advantage and one disadvantage in this essay, because you also have to write about the reasons for living in one place.)

You can read more about  developing your ideas here .

How To Write The Conclusion

In the conclusion to an IELTS Direct Questions essay, you need to do one thing:

  • summarise your main points

Do NOT write any new ideas in your conclusion. If you think of new ideas while writing your conclusion, forget them! It’s too late.

Common Mistakes in an IELTS Direct Question Essay

These are the most common mistakes made by Test Takers when writing an IELTS Direct Questions essay:

  • not reading the question carefully enough. The questions in these essay types can vary a lot, so don’t read the questions quickly. Read them carefully.
  • Writing an overly general statement about the topic in the introduction (e.g. Education is a topic of hot debate.
  • Your main ideas are not explained and illustrated enough. You need to develop all of your ideas to get a band 7 and higher.
  • Using memorised phrases (e.g. “a hot topic”, “in a nutshell”, “pros and cons”)
  • Using “research studies” as examples: examples should illustrate your ideas, not prove them. Read about  how to use examples in IELTS essays .
  • Trying to use rare or “novel” language: examiners are looking for groups of words used naturally, not rare words.

Sample IELTS Direct Question Essay Questions

In some countries, more and more people are becoming interested in finding out about the history of the house or building they live in. What are the reasons for this? How can people research this?

(Cambridge IELTS 16 Academic Test 1)

In their advertising, businesses nowadays usually emphasise that their products are new in some way. Why is this? Do you think it is a positive or negative development?

(Cambridge IELTS 16 Academic Test 2)

In some countries, owning a home rather than renting one is very important for people. Why might this be the case? Do you think this is a positive or negative situation?

( Cambridge IELTS 15 Academic Test 1 )

In many countries today, crime novels and TV crime dramas are becoming more and more popular. Why do you think these books and TV shows are popular? What is your opinion of crime fiction and TV crime dramas?

(Cambridge IELTS 15 General Training Test 1)

Model IELTS Direct Question Essays

Here is an IELTS Direction Questions Essay that I wrote in response to this task:

In many countries today, crime novels and TV crime dramas are becoming more and more popular.

Why do you think these books and TV shows are popular?

What is your opinion of crime fiction and TV crime dramas?

Stories about criminal activity, both fictional and real-life, have become increasingly popular over the last few decades. There are many possible reasons for this, but the two primary ones that I can think of are the underlying desire of people to see good overcome evil, and a fascination with criminal lifestyles.

Almost all stories about crime, whether in print or on TV, are about good people, such as detectives and law-abiding civilians, triumphing over bad people, namely criminals. We often see this in fictional detective stories, where an otherwise ordinary person uses their intellect and skill to identify evil criminal masterminds. A good example of this is Miss Marple, an elderly woman who always manages to track down and apprehend evil criminals.

A second reason is that people have a fascination with the lives of criminals. Perhaps this is to do with people’s need for escapism. One of the most popular crime dramas in the UK of the last 20 years was ‘Legend’, a dramatisation of the lives of the Kray Twins, two violent London gang leaders of the 1960s. The film, which I watched on TV, portrayed their violent behaviour, along with their opulent and chaotic lifestyles, and I do feel that people find this compelling viewing, despite how it shows evil people succeeding.

Personally, unless it is related to real-life stories, I have little interest in either crime fiction or crime drama. I find their plots too repetitive. With true crime stories, however, I can learn something about social history and psychology. Why, for example, do people turn to lives of crime? Is it simply for money, or are they motivated by power as well? And what causes people to join gangs and follow people like the Krays? These are all interesting questions.

In summary, a desire to see good triumph over evil, along with a fascination with evil, are two reasons I think underlie the popularity of crime stories, but my interest in them is mainly limited to dramatisations of real lives.

(335 words)

Read my full plan and comments for this essay.

More IELTS Direct Question Sample Essays

Share this:.

  • Click to share on Facebook (Opens in new window)
  • Click to share on WhatsApp (Opens in new window)
  • Click to share on Twitter (Opens in new window)
  • Click to share on LinkedIn (Opens in new window)
  • Click to share on Reddit (Opens in new window)
  • Click to share on Pinterest (Opens in new window)
  • Click to share on Telegram (Opens in new window)
  • Click to share on Pocket (Opens in new window)
  • Click to print (Opens in new window)
  • Click to share on Tumblr (Opens in new window)

Related Posts

direct questions essay

About the author

Charlie is a former IELTS Examiner with 25 years' teaching experience all over the world. His courses, for both English language learners and teachers, have been taken by over 100,000 students in over 160 countries around the world.

IELTS® is a registered trademark of Cambridge English Language Assessment, the British Council, and IDP Education Australia. By using this website, you agree that you fully understand that ieltscharlie.com is not affiliated, approved or endorsed by Cambridge English Language Assessment, the British Council, or IDP Education Australia.

Unit 22489, PO Box 6945, London, W1A 6US, United Kingdom

© IELTSCharlie

Privacy Overview

Discover the 7 STEPS to BAND 7 in IELTS Writing Task 2

  • Ebooks & Courses
  • Practice Tests

How to Plan & Write IELTS Double Question Essays

IELTS double question essays are also known as ‘direct question’ or ‘two questions’ essays. They are distinguished by two characteristics:

  • They have one statement with two different questions after it.
  • The questions may or may not be linked.

Here are 3 examples:

1) Fossil fuels are essential for producing electricity, powering industry and fueling transportation. However, one day we will reach a point when all the world’s fossil fuels have been depleted.

How can we conserve these resources?

What are some alternatives to fossil fuels?

2) Some parents buy their children whatever they ask for, and allow their children to do whatever they want.

Is this a good way to raise children?

What consequences could this style of parenting have for children as they get older?

3) The arts, including art, music and theatre are considered to be important in society.

Do you think the arts still have a place amongst our modern lifestyles?

Should the arts be included in the school curriculum?

In this lesson, I’m going to demonstrate step-by-step how to plan and write IELTS double question essays.

Here’s what we’ll be covering:

  • 3 Common mistakes
  • Essay structure
  • How to plan
  • How to write an introduction
  • How to write main body paragraphs
  • How to write a conclusion

Want to watch and listen to this lesson?

Click on this video.

Click the links to see lessons on each of these Task 2 essay writing topics. 

Once you understand the process, practice on past questions. Take your time at first and gradually speed up until you can plan and write an essay of at least 250 words in the 40 minutes allowed in the exam.

3 Common Mistakes

These three errors are common in IELTS double question essays.

  • Not answering both questions fully.
  • Not outlining both answers in the introduction.
  • Mistaking it for one of the other essay types.

Many students make the mistake of only answering one of the questions, or focusing more on one question than the other which leads to an unbalanced essay. Both these errors will seriously affect your score for task achievement.

You must outline everything you are going to write about in the introduction. This is your blueprint for the whole essay. I’ll show you how to do this and get your essay off to a great start.

It’s easy to mistake IELTS double question essays for one of the other four types of Task 2 essays, especially opinion or discussion essays. Each should be answered in a slightly different way.

Analysing the question properly is essential to avoiding this error. I’ll also show you how to do this and give you a simple 4 part structure for planning your essay.

Essay Structure

Let’s look at this essay structure straight away. You can use it to write any IELTS double question essay. It’s easy to learn and will enable you to quickly plan and write a high-level essay.

1)  Introduction  

  •   Paraphrase the question 
  •   Outline sentence – state your answer to both questions

2)  Main body paragraph 1 – Answer question 1

  • Topic sentence – state your answer
  • Explanation – develop the idea
  • Example – give an example

3)  Main body paragraph 2 – Answer question 2

4)  Conclusion Summarise both questions and answers

This structure will give us a well-balanced essay with 4 paragraphs.

We now need some ideas to add to the structure and we’ll have everything we need for our essay.

How To Plan IELTS Double Question Essays

Here’s the question we’re going to be answering in our model essay followed by the 3 steps of the planning process.

Fossil fuels are essential for producing electricity, powering industry and fueling transportation. However, one day we will reach a point when all the world’s fossil fuels have been depleted.

  • Analyse the question
  • Generate ideas
  • Identify vocabulary

# 1  Analyse the question

This is an essential step in the planning process and will ensure that you answer the question fully. It’s quick and easy to do. You just need to identify 3 different types of words:

  • Topic words
  • Other keywords
  • Instruction words

Topics words  are the ones that identify the general subject of the question and will be found in the statement part of the question.

Fossil fuels  are essential for producing electricity, powering industry and fueling transportation. However, one day we will reach a point when all the world’s  fossil fuels  have been depleted.

So, this question is about ‘ fossil fuels ’.

Many people will do this first step of the process and then write about the topic in general. This is a serious mistake and leads to low marks for task achievement.

Now that we know what the general topic is, we need to understand exactly what aspect of fossil fuels we're being asked to write about.

The  other keywords  in the question tell you the specific things you must write about. For IELTS double question essays, these will often be in the instructions, that is, the actual questions.

How can we  conserve  these resources?

What are some  alternatives  to fossil fuels?

By highlighting these words, it’s easy to identify the topics. Your essay must only include ideas relevant to these ideas.

The  instruction words  are the questions themselves. These tell you exactly what type of information is required and each will become the topic for one of the two main body paragraphs.

The first body paragraph will answer the first question (How?) and the second body paragraph will answer the second question (What?).

# 2  Generate ideas

The next task is to generate some ideas to write about.

There are several different ways to think up ideas. I cover them fully on the  IELTS Essay Planning  page.

We’re going to use the ‘friends technique’. This is the method I prefer as it allows you to take a step back from the stress of the exam situation and think more calmly.

Here’s how it works. Imagine that you are in a casual conversation with a friend over a cup of coffee and they ask you this question. What are the first thoughts to come into your head? Plan your essay around these ideas.

Doing this will help you to come up with simple answers in everyday language rather than straining your brain to think of amazing ideas using high-level language, which isn’t necessary.

You might want to try this yourself before reading on for my ideas.

Here are my ideas as I thought of them:

How can we conserve these resources? 

  • Become more energy conscious & more energy efficient
  • Use more renewable energy sources – solar panels
  • All new homes should be built with solar panels on
  • Use car less – walk, cycle, public transport, only travel when really necessary
  • Energy-efficient light bulbs
  • Solar power
  • Wave energy
  • Tidal energy
  • Biomass energy
  • Geothermal energy

Don’t spend long on this as you only need one or two ideas.

There is so much to write about this topic that we have to be very careful we don’t try to include too many different ideas and just end up with a list for each question rather than a well-developed essay.

Choose one main idea for each part of the question. My advice on making your selection is to choose ideas that you can quickly think of examples for.

Here are my choices:

  • Use car less – walk, cycle, public transport
  • Natural forces – solar & wind power, wave & tidal energy

We’re almost ready to start writing our IELTS double question essay but first, we have one other small task to do.

# 3  Vocabulary

During the planning stage, quickly jot down some vocabulary that comes to mind as you decide which ideas you are going to write about, especially synonyms of key words. This will save you having to stop and think of the right language while you’re writing.

For the ideas I’ve chosen, useful words will include:

  • sustainable  
  • renewable energy
  • energy-efficient

With that done, we can focus on the first paragraph of the essay – the introduction.

How To Write an Introduction

The best introductions to IELTS double question essays have a simple 2 part structure:

1)   Paraphrase the question

2)   Outline sentence – state your answer to both questions

  • Have 2-3 sentences
  • Be 40-60 words long
  • Take 5 minutes to write

1)  Paraphrase the question

Start your introduction by paraphrasing the statement part of the question.

Question statement:

Paraphrased question:  

The world is currently reliant on oil, coal and natural gas for the majority of its energy requirements but there will come a time when these run out.

We are simply saying the same thing in a different way and using different vocabulary.

2)  Outline statement

Now we need to add an  outline statement  where we outline the two main points that we’ll cover in the rest of the essay, that is, the answers to the two questions.

We need to be very specific about what we are going to write about.

Here's a reminder of the ideas I’ve chosen to answer the two questions:

  • Natural forces –solar & wind power, wave & tidal energy

Outl ine statement:  

This essay will discuss how we can help to prevent our non-renewable resources from becoming depleted by using our cars less frequently and it will name some natural forces that can be harnessed to generate power.

Note my use of synonyms to replace key words in the question. You don’t have to replace every key word but do so where possible whilst ensuring that your language sounds natural.

So, let’s bring the two elements of our introduction together.

     Introduction

direct questions essay

This introduction achieves three important functions:

  • It shows the examiner that you understand the question.
  • It acts as a guide to the examiner as to what your essay is about.
  • It also helps to keep you focused and on track as you write.

The two ideas in your introduction will become your two main body paragraphs.

Main body paragraph 1  –  Use car less – walk, cycle, public transport

Main body paragraph 2  –  Renewable energy / natural forces – solar & wind power, wave & tidal energy

How To Write Main Body Paragraphs

Main body paragraphs in IELTS double question essays should contain 3 things:

  • Explanation –  develop the idea

Main Body Paragraph 1  – Answer question 1

The  topic sentence  summarises the main idea of the paragraph. That’s all it needs to do so it doesn’t have to be complicated.

It plays an important role in ensuring that your ideas flow logically from one to another. It does this by acting as a signpost for what is to come next, that is, what the paragraph will be about.

If you maintain a clear development of ideas throughout your essay, you will get high marks for task achievement and cohesion and coherence.

We’ll now take the idea for our first main body paragraph and create our topic sentence.

Main body paragraph 1  –  Use car less – walk, cycle, public transport, only travel when really necessary

Topic sentence:  

Conserving energy is a responsibility of every individual and an important way in which we can all do our bit is to use more energy-efficient means of transport. 

Next, we must write an  explanation sentence  that develops the idea.

Explanation sentence: 

The easiest way to do this is to leave the car at home and walk or cycle to our destination if it isn’t too far away, or take public transport for longer journeys. Another way to reduce our fuel consumption is to car share.

Finally, we add an  example  to support our main point. If you can’t think of a real example, it’s fine to make one up, as long as it’s believable. The examiner isn’t going to check your facts. Alternative, you could add another piece of information to support your idea but an example is better.

Example sentence:

Whenever my friends and I get together for coffee, we agree to meet up at a café that we can each get to without having to drive our cars there. We usually go on foot or ride our bikes. If everyone made small decisions like this, it would make a real difference.

That’s the 3 parts of our first main body paragraph complete. Here’s the finished paragraph.

direct questions essay

We now follow the same process for our second main body paragraph.

Main Body Paragraph 2  – Answer question 2

Again, we’ll now take the idea I’ve chosen for this paragraph and create our topic sentence.

Main body paragraph 2  –  Renewable energy / natural forces –   solar & wind power, wave & tidal energy

Topic sentence:

The most sustainable alternatives to fossil fuels are the generation of power from natural forces such as the sun, wind and oceans.

Now for the  explanation  where we expand on this idea.

Explanation sentence:

S olar and wind power are already widely used across the world but it is wave power and tidal energy that have the greatest untapped potential to provide for our energy needs in the future.

Finally, an  example  to support our main point.

A report recently commissioned in the United Kingdom estimates that tidal energy could meet as much as  20% of the UK’s current electricity demands once the technology being developed is operational. Wave energy converters are expected to prove equally successful in the long-term.

That’s the 3 parts of our second main body paragraph complete. Here’s the finished paragraph.

direct questions essay

Now we need a conclusion and our IELTS double question essay is done.

How To Write a Conclusion

The conclusion is a summary of the main points in your essay and can often be done in a single sentence. It should never introduce new ideas.

If you're below the minimum 250 words after you’ve written your conclusion, you can add a prediction or recommendation statement.

Our essay is already over the minimum word limit so we don’t need this extra sentence but you can learn more about how to write a prediction or recommendation statement for IELTS double question essays on the  Task 2 Conclusions  page.

The conclusion is the easiest sentence in the essay to write but one of the most important.

A good conclusion to an IELTS double question essay will:

  • Neatly end the essay
  • Link all your ideas together
  • Sum up your argument or opinion
  • Answer the question

If you achieve this, you’ll improve your score for both task achievement and cohesion and coherence which together make up 50% of the overall marks. Without a conclusion, you’ll score below band 6 for task achievement.

You can start almost any final paragraph of an IELTS double question essay with the words:

  • In conclusion

        or

  • To conclude

Now all you need to do is briefly summarise the main ideas into one or two sentences.

Here’s a top tip . Go back and read the introduction to the essay because this is also a summary of the essay. It outlines what you are going to write about.

To create a great conclusion, you simply have to paraphrase the introduction.

Introduction:

Here is the same information formed into a conclusion:

direct questions essay

That’s it. We’ve completed our essay. Here it is with the 4 paragraphs put together.

Finished IELTS double question essay.

direct questions essay

     (351 words)

Go through this lesson as many times as you need to in order to fully understand it and put in lots of practice writing IELTS double question essays from past exam questions. Practice is the only way to improve your skills.

Would you prefer to share this page with others by linking to it?

  • Click on the HTML link code below.
  • Copy and paste it, adding a note of your own, into your blog, a Web page, forums, a blog comment, your Facebook account, or anywhere that someone would find this page valuable.

Like this page?

More help with ielts double question essays & other task 2 essays.

IELTS Writing Task 2  – T he format, the 5 question types, the 5 step essay writing strategy & sample questions. All the key information you need to know.

The 5 Types of Task 2 Essay   – How to recognise the 5 different types of Task 2 essays. 15 sample questions to study and a simple planning structure for each essay type.

Understanding Task 2 Questions  – How to quickly and easily analyse and understand IELTS Writing Task 2 questions.

How To Plan a Task 2 Essay  – Discover why essay planning is essential & learn a simple 4 step strategy, the 4 part essay structure & 4 methods of generating ideas.

How To Write a Task 2 Introduction  – Find out why a good introduction is essential. Learn how to write one using a simple 3 part strategy & discover 4 common mistakes to avoid.

How To Write Task 2 Main Body Paragraphs  – Learn the simple 3 part structure for writing great main body paragraphs and also, 3 common mistakes to avoid. 

How To Write Task 2 Conclusions  – Learn the easy way to write the perfect conclusion for a Task 2 essay. Also discover 4 common mistakes to avoid.

Task 2 Marking Criteria  – Find out how to meet the marking criteria in Task 2. See examples of good and poor answers & learn some common mistakes to avoid.

The 5 Task 2 Essay Types:

Step-by-step instructions on how to plan & write high-level essays. Model answers & common mistakes to avoid.

   Opinion Essays

   Discussion Essays

  Problem Solution Essays

  Advantages & Disadvantages Essays

  Double Question Essays

Other Related Pages

IELTS Writing Test  – Understand the format & marking criteria, know what skills are assessed & learn the difference between the Academic & General writing tests.

  • IELTS Writing
  • Double Question Essays
  • Back To Top

 * New * Grammar For IELTS Ebooks

direct questions essay

$9.99 each       Full Set   Just   $ 23.97

Find Out More >>

IELTS Courses

direct questions essay

Full details...

direct questions essay

IELTS Writing Ebook

direct questions essay

Discount Offer

$7 each       Full Set Just   $ 21

direct questions essay

Find out more >>

Testimonials

“I am very excited to have found such fabulous and detailed content. I commend your good work.”  Jose M.

“Thanks for the amazing videos. These are ‘to the point’, short videos, beautifully explained with practical examples."  Adari J.

"Hi Jacky, I bought a listening book from you this morning. You know what? I’m 100% satisfied. It’s super helpful. If I’d had the chance to read this book 7 years ago, my job would be very different now."  Loi H.

"Hi Jacky, I recently got my IELTS results and I was pleased to discover that I got an 8.5 score. I'm firmly convinced your website and your videos played a strategic role in my preparation. I was able to improve my writing skills thanks to the effective method you provide. I also only relied on your tips regarding the reading section and I was able to get a 9! Thank you very much." Giano

“After listening to your videos, I knew I had to ditch every other IELTS tutor I'd been listening to. Your explanations are clear and easy to understand. Anyways, I took the test a few weeks ago and my result came back: Speaking 7, listening 9, Reading 8.5 and Writing 7 with an average band score of 8. Thanks, IELTS Jacky." Laide Z.

      Contact

      About Me

      Site Map

      Privacy Policy

      Disclaimer

IELTS changes lives.

Let's work together so it changes yours too.

Copyright  © 2024     IELT Jacky     

All Right Reserved

IELTS is a registered trademark of the University of Cambridge, the British Council, and IDP Education Australia. This site and its owners are not affiliated, approved or endorsed by the University of Cambridge ESOL, the British Council, and IDP Education Australia.

TED IELTS

  • A Beginner’s Guide to IELTS
  • Common Grammar Mistakes [for IELTS Writing Candidates]

Writing Correction Service

  • Free IELTS Resources
  • Practice Speaking Test

Select Page

IELTS Writing Task 2: Two-Part Questions

Posted by David S. Wills | Jul 18, 2018 | IELTS Tips , Writing | 1

IELTS Writing Task 2: Two-Part Questions

Today we’re going to look at an IELTS writing task 2 question that often makes students a little worried: the two-part question . This is also sometimes referred to as “the direct question” as it is more direct than other types of writing questions, such as advantages and disadvantages or causes and solutions .

In this lesson, I will show you what this question looks like and explain how to answer it.

Table of Contents

What is a two-part question, two-part question examples, analysing the question, structuring a two-part question essay, sample answer.

In IELTS writing task 2, there are different sorts of questions you may be asked. One of them is called the two-part question (or sometimes “the direct question”). It is most commonly called a two-part question because it contains two distinct questions.

Of course, some other questions also contain two parts. For example, a problem and solution essay is two parts. However, what we mean by a “two-part question” is one that contains two questions . The reason this is sometimes called a “direct question” task is that the questions themselves are more direct than other IELTS writing task 2 question types, which instruct candidates to explore an idea. This is rather vague , whereas the two-part question is very specific .

To understand this idea better, let’s look at an example two-part question:

In education and employment, some people work harder than others. Why do some people work harder? Is it always a good thing to work hard?

Ok, the first thing that you may notice is that there are three parts to this question! However, the first part is actually a lead-in statement. It is not particularly important. The parts that you need to address in your essay are the two questions. This is why some people say “two-part question” and others say “direct question”.

Here is another example:

Happiness is often considered difficult to define. Why is this? What factors determine happiness?

Here the first question contains a pronoun, “this”, which refers back to the idea in the first sentence. In other words, the first question is “Why is it often considered difficult to define happiness?”

Finally, here is one more example question:

Success is often measured by wealth and material possessions. Do you think wealth is the best measure of success? What makes a successful person?

As you can see by now, each question has three parts: an introductory statement and two individual (but related) questions. You will see this exact format used with numerous IELTS topics .

Sometimes the lead-in statement will be very long, and sometimes the questions are quite closely linked. Other times the statement may be short or the questions ask quite different things.

How to Answer Two-Part Questions

The first thing to remember is: DON’T PANIC! Two-part questions often make students a bit nervous, but really they are not so bad. They are no more difficult to answer than the other IELTS task 2 question types.

In fact, this type of question may even be the easiest one to answer!

Although the questions themselves are sometimes a bit challenging, they are direct questions . This means that it is less likely you will stray off-topic while writing. In other words, your task is actually more straightforward than it would be with an agree/disagree question, for example.

Let’s look at an example question so we can analyse it. This is the first thing you should do in any IELTS writing task 2 essay, and it’s important to spend a minute or two thinking carefully about it.

Some parents buy their children whatever they ask for, and allow their children to do whatever they want. Is this a good way to raise children? What consequences could this style of parenting have for children as they get older?

We can take three steps to analyse this question.

  • Let’s look at the first sentence: What is it about? It is about permissive parents – ie parents who give their children too much (or who give in to their child’s demands) and let them do too many things.
  • Next, the first question contains a pronoun (“this”) which refers to the ideas in the first sentence, and asks if it’s good or not.
  • The second question talks about the consequences – not for the parents, but for the children.

I have deliberately chosen a slightly difficult question here in order to show you how to answer it. Usually, the questions are a little more direct and obvious, but here you need to consider the three sentences in order, making sure to understand each fully.

In a situation like this, if you make an incorrect assumption about the first or second parts of the sentence, it may cause a serious lack of coherence and cohesion , in addition to poor task achievement .

For example, if you thought it was just about children not being made to study enough, you might write the next paragraph all about a lack of studying and the following one may come back closer to the correct idea because of the more specific question. This would look bad, indicating a lack of unity in your writing.

Thankfully, it is not difficult to create a perfect two-part essay structure. In fact, it is very easy!

Here is a basic essay template:

Explain the main idea
Give an
Body paragraph #1Answer the first question
Body paragraph #2Answer the second question
Summarise your essay briefly

In a previous article, I talked about whether to use a 4 or 5 paragraph essay . This is one case when you definitely want to use a four-paragraph essay.

Here’s the video, if you want to review it:

I have lots more posts on the topic of IELTS essay structures. Here are a few: 

  • IELTS Writing Task 2 Essay Structures
  • Planning IELTS Writing Task 2 Structure
  • How to Structure a Paragraph

In this section, I will give you my sample answer to the above question, using the basic four-paragraph structure that I mentioned. Obviously, my ideas will be different to yours, but the structure which I outlined can be used 100% of the time.

In today’s society, some parents are becoming increasingly permissive. They do not impose sufficient discipline on their children, and in some cases buy them too many things. This essay will explore why this is not a good way to raise children and why it will have negative impacts upon them in future. While it is understandable that parents want to give their children expensive toys and let them run freely in the streets, this is actually not really an appropriate method of parenting. Having too many toys encourages children to be materialistic and does not offer them the same change to develop social skills like sharing. Furthermore, when children have asked for the toys, it gives them a sense of entitlement and even power over their parents. As for giving children too much freedom, there are obviously a great many dangers in this world from which they need to be kept safe. Children also need rules and boundaries to encourage them to develop into mature and responsible adults. If parents insist on this permissive style of parenting, their children will grow up with very different values from those of stricter parents. Children who were never forced to study will end up with poorer grades in school, and those who were given everything they wanted as children will expect everything to come easily as adults. In short, they will lead difficult and disappointing lives, in contrast with what their parents hoped. In conclusion, although it is tempting to give children whatever they want, parents ought to set rules and boundaries, and to be careful with how they reward children. If parents fail to impose a basic level of discipline, children may grow up with a poor attitude that will cause them and others to suffer.

There are really no words or phrases that are unique to the two-part question essay. Just use regular academic English as with any other IELTS task 2 question, and of course stick to the topic.

For my essay, I used the word “permissive.” It means allowing too much and not setting enough rules. You could use similar words like “liberal,” “easygoing”, “live and let live”, and so on, although they vary slightly in precise meaning.

Here are some more useful terms that you can use to talk about parents:

direct questions essay

As always, remember to avoid IELTS phrases and other cliches. Learn new vocabulary by topic and in collocations rather than isolation.

Tips for Two-Part Questions

Finally, a few tips to remember:

  • Practice this question type often before the exam – it’s pretty common!
  • Make sure you understand each part of the question before answering.
  • Always use the four-paragraph essay structure.
  • Work on idea generation at home in order to come up with good, on-topic answers.

I made this article into a short video. Please give it a ‘LIKE’ and subscribe to the channel if you find it useful.

About The Author

David S. Wills

David S. Wills

David S. Wills is the author of Scientologist! William S. Burroughs and the 'Weird Cult' and the founder/editor of Beatdom literary journal. He lives and works in rural Cambodia and loves to travel. He has worked as an IELTS tutor since 2010, has completed both TEFL and CELTA courses, and has a certificate from Cambridge for Teaching Writing. David has worked in many different countries, and for several years designed a writing course for the University of Worcester. In 2018, he wrote the popular IELTS handbook, Grammar for IELTS Writing and he has since written two other books about IELTS. His other IELTS website is called IELTS Teaching.

Related Posts

Skimming and Scanning Practice – UFOs

Skimming and Scanning Practice – UFOs

April 2, 2016

Answering the Question – Task Achievement for IELTS Writing

Answering the Question – Task Achievement for IELTS Writing

May 7, 2019

Can Vocabulary and Grammar Mistakes Impact Task Response?

Can Vocabulary and Grammar Mistakes Impact Task Response?

September 9, 2022

Paraphrasing and IELTS: Is it Really Important?

Paraphrasing and IELTS: Is it Really Important?

March 9, 2021

Mariam

could someone evaluate this and give the band score it deserves? here is my writing of the given topic….. In education and employment, some people work harder than others. 1. Why do some people work harder? 2. Is it always a good thing to work hard?

ans : It is often seen that some people put in more effort than others in the field of education and employment . The possible reasons could be due to the workload pressure or genuine passion .While passion for accomplishing task is a good thing, working hard due to constant pressure and other circumstances could be a problem . This essay explores the potential reasons as to why some work more than others and also discusses whether it is good to do so or not.

While students work hard due to their caliber and interest in studies working people work hard due to circumstances .Although there are a few students who study due to constant nagging by mentors or parents, most of them develop a genuine interest towards the subjects. Employers, on the other hand, work up to their innate potential to make both ends meet. For example, some workers work overtime that is for longer hours in order to earn higher salary or during the weekends to cope with the pending works . Sometimes they complete their tasks in advance in case they have any upcoming projects or plans outside the workplace to compensate for the missing working days.

Whether it is a good thing or a bad thing completely depends on the situation. Any work, no matter how important and urgent, if done against will and under high pressure will never give good outcomes . Instead it will lead to anxiety and stress if not accomplished on time. But if one is passionate and hungry for success and enjoys pursuing their interest, it will undoubtedly bear fruits and their hard work will eventually pay off.

All things considered, hard work is the key to success with respect to both education and employment. But it is important to make a note that one should have a great deal of passion and sincerity to achieve success and should not be forced to work due to helplessness or under pressure.

Leave a reply Cancel reply

Your email address will not be published. Required fields are marked *

This site uses Akismet to reduce spam. Learn how your comment data is processed .

Download my IELTS Books

books about ielts writing

Recent Posts

  • Exams vs Continual Assessement [Model Essay]
  • British vs American Spelling
  • How to Improve your IELTS Writing Score
  • Past Simple vs Past Perfect
  • Complex Sentences

ielts writing correction service

Recent Comments

  • Raza Aamir on IELTS Speaking Partners
  • David S. Wills on Writing Correction Service
  • kenji on Writing Correction Service
  • Pop Pop on Straw No More, by Molly Steer
  • Francisca on Adverb Clauses: A Comprehensive Guide
  • Lesson Plans
  • Model Essays
  • TED Video Lessons
  • Weekly Roundup

IELTS Exams logo

Direct Question Essay IELTS 2024: Tips, Structure, Sample

Looking for Direct Question Essay IELTS tips Click to learn how to plan, recognise prompts get samples for a successful IELTS essay!

9/26/2023 2 min read

direct questions essay

In the world of standardized English language proficiency tests, the IELTS (International English Language Testing System) is a formidable challenge for many. One of its key components, the Direct Question Essay, is often a source of anxiety for test-takers. But fear not! In this comprehensive guide, we will unravel the intricacies of the Direct Question Essay in IELTS, offering valuable tips and insight into its structure and providing sample questions with model answers.

Understanding the Direct Question Essay

What is a direct question essay in ielts.

The Direct Question Essay, or the Opinion Essay, is an essay task in the IELTS writing section. It presents a clear and straightforward question that requires a direct response. Test-takers are expected to provide their opinion, supported by relevant arguments and examples.

Why is it Important?

The Direct Question Essay assesses a candidate's ability to express opinions coherently, develop arguments logically, and use appropriate vocabulary and grammar. It's crucial to perform well in this task to achieve a high band score in the IELTS writing module.

Tips for Success

Analyze the Question Carefully

Before diving into your response, take a moment to understand the question thoroughly. Identify keywords and phrases that define the scope of your answer.

Plan Your Response

Outline your essay with a clear introduction, body paragraphs, and a conclusion. This structure helps organize your thoughts and ensures a coherent essay.

Provide a Clear Opinion

In the introduction, state your opinion on the given topic. Make it concise and specific, setting the tone for the rest of the essay.

Support Your Opinion

Back up your opinion with reasons and examples. Use real-life experiences or hypothetical situations to illustrate your points.

Use Formal Language

While maintaining a conversational tone, remember that IELTS essays require formal language. Avoid slang and contractions.

Vary Sentence Structure

Enhance the readability of your essay by using a variety of sentence structures. Combine simple, compound, and complex sentences for better flow.

Proofread and Edit

Allocate time for proofreading. Check for spelling, grammar, and punctuation errors. Ensure your essay is polished and error-free.

Structure of a Direct Question Essay

The structure of a Direct Question Essay typically consists of:

Introduction

Introduce the topic and provide a clear opinion.

Body Paragraphs

Each paragraph should focus on a single supporting point.

Provide examples and evidence to strengthen your arguments.

Summarize your main points and restate your opinion.

Sample Questions and Model Answers

Question 1:.

"Do you agree or disagree with the statement that technology has made people more isolated?"

Model Answer:

Technology has made people more isolated. In today's digital age, individuals are often engrossed in their smartphones and computers, limiting face-to-face interactions. For instance, while connecting people virtually, social media platforms have also contributed to a decline in genuine personal connections.

Question 2:

"Is it better to study alone or in a group?"

Studying in a group yields better results. When you study with others, you can share ideas, clarify doubts, and gain different perspectives. Moreover, group study sessions encourage discipline and time management.

Mastering the Direct Question Essay in IELTS requires practice and a clear understanding of its structure and requirements. By following these tips and reviewing sample questions and model answers, you can boost your confidence and perform exceptionally well in this challenging task.

Subscribe newsletter

  • Skip to primary navigation
  • Skip to main content
  • Skip to primary sidebar
  • Skip to footer

IELTS Advantage

IELTS Advantage

IELTS Preparation Courses

IELTS Writing Task 2 Essay Structures

The four most common IELTS writing Task 2 questions are: Opinion, Advantages and Disadvantages, Problem and Solution Discussion

IELTS Task 2 Essay Structures

Knowing how to structure your IELTS Writing Task 2 essay is an essential skill that can make the difference between getting and not getting the band score you deserve. With that in mind, we have outlined the most common IELTS Writing Task 2 structures below.

direct questions essay

Nearly all of my Task 2 essays follow this basic structure: The sentences you put in each paragraph will depend on what type of question you get.

The five most common IELTS Writing Task 2 questions are:

  • Opinion (Agree or Disagree)
  • Advantages and Disadvantages
  • Problem and Solution
  • Discussion (Discuss both views)
  • Two-part Question

Below I will outline examples and a structure approved by experienced IELTS teachers and examiners for each type of question. This will help you write a clear, coherent answer and hopefully boost your IELTS band score. I also include an example answer for each type of question so you can see the structure in a real essay.

Please note that these are general structures and may vary slightly depending on the question.

Please also note that no ‘one’ Task 2 essay structure will get you a high score. There are many types of structures that can get you a high score. These are just some I think are effective and easy to learn. 

Please visit the lessons below for more detailed guidance on each type of question. I have provided a link at the end of each section.

direct questions essay

Opinion Questions (Agree or Disagree) 

Typical Question Words –

What is your opinion?

Do you agree or disagree?

To what extent do you agree or disagree?

Direct question.

Example Question –

Some people believe that unpaid community service should be compulsory in high school programmes (for example, working for a charity, improving the neighbourhood or teaching sports to younger children).

Essay Structure 

Introduction 

1- Paraphrase Question

2- Give your opinion and outline the main ideas.

Main Body Paragraph 1 

1- Topic Sentence

2- Explain Topic Sentence

Main Body Paragraph 2

Conclusion 

1- Summary of main points and opinion

Student Sample Answer

It is argued that volunteering should be made part of the school curriculum. This essay agrees with that suggestion completely because it help pupils develop soft skills and helps them gain much-needed work experience.

Education should not be limited to strictly academic pursuits, and those in education should also develop life skills, such as teamwork, empathy and self-discipline, and one of the best ways to hone these aptitudes is through community service. Serving those less fortunate than ourselves teaches us many lessons, including how to work with people from other backgrounds and the value of hard work, thus enabling us to hone these skills before becoming an adult. For example, many young people from wealthier countries take a gap year and help those less fortunate than themselves to increase their gratitude for what they have and improve their work ethic.

Many colleges and companies are also increasingly looking for this type of experience. Most school leavers have the same grades, and charitable work can help set you apart from other students when making college applications. For example, Cambridge and Oxford receive thousands of applications from straight-A students yearly and can only accept a small percentage of applicants. What you have done outside the classroom often differentiates you from everyone else and gets you that coveted spot.

In conclusion, teenagers should be made to partake in unpaid work as part of their schooling because it will help them learn things they wouldn’t ordinarily learn from their teachers, and it will also boost their chances of getting into third-level education.

For more detail on how to answer agree or disagree questions, please visit our opinion essay lesson . 

Need help writing essays like this? Check out our ESSAY CORRECTION SERVICE .

Advantages and Disadvantages Questions

Typical Question Words 

Discuss the advantages and disadvantages.

What are the advantages and disadvantages?

Example Question

Technology is being used more and more in education.

Essay Structure

2- Outline Main Points

Main Body Paragraph 1

1- State Two Advantages

2- Expand/Explain First Advantage

3- Expand/Explain Second Advantage

1- State Two Disadvantages

2- Expand/Explain First Disadvantage

3- Expand/Explain Second Disadvantage

direct questions essay

1- Summary of Main Points

Student Sample Answer 

It is argued that technology plays an ever-increasing role in schools and universities. Increased access to information and student freedom are the main advantages, whereas dependency on technology and decreasing levels of face-to-face contact are the main disadvantages.

Access to more information and student autonomy are the principal advantages of increasing the use of electronic devices in education. With the internet, students can access all the information available about any topic, regardless of what books and other resources are available in the school. Furthermore, students can focus on whatever topic or subject they want and study it in depth. A prime example of this is the number of online university courses available to students, covering a myriad of subjects that, up until recently, were unavailable to most learners. This has resulted in more people studying third-level degrees than ever before at a pace and schedule that suits them.

The main disadvantages associated with the increasing use of technology in education are the dependency on this technology and the decrease in face-to-face interaction between students. With many students now using the internet as their primary source of information, they often struggle to use other academic resources to find what they’re looking for. As well as this, students spend more time looking at computer screens by themselves than interacting with each other, which is thought to lead to lower levels of emotional intelligence. For instance, the recent explosion in smartphone use has been at the expense of genuine human interaction. This results in soft skills, such as verbal communication and empathy, being affected.

In conclusion, the benefits technology brings to education, such as unrestricted access to information and student autonomy, must be weighed against the drawbacks, such as dependency on this technology and the negative effects on human interaction.

For more detail on how to answer advantage and disadvantage questions, please visit our  advantage and disadvantage lesson . 

Discuss Both Views Question (Discussion Essay)  

direct questions essay

Discuss both points of view and give your opinion.

Example Question 

Technology is being used more and more in education. Some people say that this is a positive trend, while others argue that it is leading to negative consequences.

Discuss both sides of this argument and then give your own opinion.

1- Paraphrase Question and/or state both viewpoints.

2- Thesis Statement

3- Outline Sentence

1- State first viewpoint

2- Discuss first viewpoint

3- Reason why you agree or disagree with viewpoint

4- Example to support your view

1- State second viewpoint

2- Discuss second viewpoint

Sentence 1- Summary

Sentence 2- State which one is better or more important

There is an ever-increasing use of technology, such as tablets and laptops, in the classroom. It is often argued that this is a positive development, whilst others disagree and think it will lead to adverse ramifications. This essay agrees that an increase in technology is beneficial to students and teachers.

The Internet has provided students with access to more information than ever before. This has allowed learners to research and learn about any subject at the touch of a button. It is therefore agreed that technology is a very worthwhile tool for education. Wikipedia is a prime example, where students can type in any keyword and gain access to in-depth knowledge quickly and easily.

However, many disagree and feel that technology deprives people of real human interaction. Human interaction teaches people valuable skills such as discourse, debate and empathy. Without these soft skills, many people find it difficult to become successful in work and their personal lives. Despite this, human interaction is still possible through the internet, and this essay disagrees that technology should be dismissed for this reason. For instance, Skype and Facebook allow people to interact in ways that were never before possible.

While the benefits of technology, particularly the internet, allow students to tap into limitless sources of information, some still feel that people should be wary of this new phenomenon and not allow it to curb face-to-face interaction. However, as long as we carefully consider the importance of human interaction in education, the educational benefits are clearly positive.

For more detail on how to answer discussion questions please visit our  discussion essay lesson . 

Problem and Solution Questions

direct questions essay

Problem and solution.

Cause and solution.

Students are becoming more and more reliant on technology.

What are some of the problems associated with reliance on computers, and what are some of the possible solutions?

2- Outline Sentence

1- State Problems

2- Explain First Problem

3- Explain Second Problem

4- Example of Second Problem

1- State Solutions

2- Explain First Solution

3- Explain Second Solution

4- Example of Second Solution

Learners are becoming increasingly dependent on technology, such as the Internet and mobile devices. This essay believes the main problems associated with dependence on computers are the lack of original thought and copying original work from others and suggests critical thinking classes and writing analysis software as the most viable solutions.

The principal problems with over-reliance on technology are people being unable to think for themselves and plagiarism. With access to so much information, students often rely on other people’s opinions instead of forming their own. As well as this, they often use search engines to answer a question and copy the text from a website rather than thinking about the question. This practice is prohibited in schools and universities and stunts students’ intellectual development because they will never truly think for themselves, which is what university is supposed to be for. For example, many teachers complain that students copy web pages straight from Wikipedia word for word rather than giving a reasoned answer to their questions.

Solutions to these worrying problems are special classes to focus on critical thinking and teachers using anti-plagiarism software to detect copying. If teachers create situations where students have to infer meaning and express opinions based on a small amount of information, this will ensure that students have an opportunity to develop these skills. Also, if students know that their assignments are being checked for plagiarism, this will be enough to deter them from doing so. For instance, many universities already use this kind of software to scan coursework for plagiarism, and it could be extended to include all homework by learners in both secondary and tertiary education.

In conclusion, the main problems with the overuse of technology in education are the lack of original thought and plagiarism. These can be solved through special classes that teach students analytical skills and plagiarism detection software.

For more detail on how to answer problem and solution questions please visit our  problem and solution lesson . 

Two-Part Questions

direct questions essay

There will normally be a statement, and they will then ask you to answer separate questions.

As most people spend a major part of their adult life at work, job satisfaction is an important element of individual wellbeing.

What factor contributes to job satisfaction?

How realistic is the expectation of job satisfaction for all workers?

2- Outline Sentence (mention both questions)

1- Answer first question directly

2- Explain why

3- Further explain

1- Answer second question directly

As most adults spend most of their time at work, being content with your career is a crucial part of a person’s health and happiness. This essay will first suggest fair pay as a key element leading to job satisfaction, and it will then state that it is not very likely that everyone can be happy with their job.

The most important thing that satisfies someone at work is being compensated fairly. If those more senior than you respect you as a person and the job you are doing, then you feel like you are valued. A fair salary and benefits are important marks of respect, and if you feel you are being underpaid, you will either resent your bosses or look for another job. These two factors came top of a recent job satisfaction survey conducted by Monster.com, which found that 72% of people were pleased with their current role if their superiors regularly told them they were appreciated.

With regard to the question of happiness for all workers, I think this is and always will be highly unlikely. The vast majority of people fail to reach their goals and end up working in a post they don’t really care about in return for a salary. This money is just enough to pay their living expenses which often means they are trapped in a cycle of disenchantment. For example, The Times recently reported that 89% of office workers would leave their jobs if they did not need the money.

In conclusion, being satisfied with your trade or profession is an important part of one’s well-being, and respect from one’s colleagues and fair pay can improve your level of happiness; however, job satisfaction for all workers is an unrealistic prospect.

Can I get a band 8 or 9 following these structures? 

Nobody can give you a Task 2 IELTS structure that guarantees high scores. Your score is dependent on how good your grammar and vocabulary are and how well you answer the question. A good structure will help you answer the question to some extent and boost your score for coherence and cohesion, but you must use relevant ideas and use these ideas well to answer the question.

You can see how my student scored a Band 8.5 in IELTS Writing here:

direct questions essay

Next Steps 

We hope you found those IELTS Writing Task 2 structures useful. Looking for some more sample questions? Here are over 100 sample questions from past exam papers.

If you would personalised feedback and guidance until you get the score you need, you can join the Waiting List for my VIP Course here.

' src=

About Christopher Pell

My name is Christopher Pell and I'm the Managing Director of IELTS Advantage.

I started IELTS Advantage as a simple blog to help 16 students in my class. Several years later, I am very humbled that my VIP Course has been able to help thousands of people around the world to score a Band 7+ in their IELTS tests.

If you need my help with your IELTS preparation, you can send me an email using the contact us page.

IELTS Direct Question Essay Model Answer: Education

If you would like to learn how to structure a direct question essay please click the button below: How to structure a direct question essay

If you would like to purchase a 29  page PDF download that is easy to read and print out please take a look at the bookshop >

Some people think that children should be homeschooled when they are very young while others think it is better for them to attend a kindergarten. Which do you think is better?

The following video will give you contains sample Essay in audiovisual format.

Instructor Feedback on IELTS Direct Question Essay: Education

Lexical Resource  – There is evidence of a wide range of vocabulary, with no errors in the text.

Want to check more Direct Question Essay Samples? Check the IELTS Direct Question Essay Examples

The best way to keep up to date with posts like this is to like us on Facebook , then follow us on Instagram and Pinterest . If you need help preparing for the IELTS Test, join the IELTS Achieve Academy and see how we can assist you to achieve your desired band score. We offer an essay correction service, mock exams and online courses.

Related Posts

Ielts positive/negative essay sample 5 – economics, how to write a direct question essay, leave a comment cancel reply.

direct questions essay

Best Selling

  • Basics of spoken English
  • 1:1 practice sessions for fluency
  • App (unlimited offline practice)
  • Group Discussion Mock Interviews (on demand & chargeable)
  • Group Discussion
  • Mock Interviews (on demand & chargeable)

Super Value

Ielts writing material, direct question essays.

direct questions essay

Learn how to write a Direct Question essay in IELTS Writing task 2. you’ll have to write an essay of at least 250 words on a topic.
Beginning Paragraph

There should always be a paragraph at the beginning of the essay. You might begin this paragraph by briefly restating the query and introducing the subject matter of the body paragraphs.

Body Section 1

Here, you can respond to the query and discuss the scenario that it suggests. And talk about the initial query (if there are more than 1 question). Also provide examples.

Body Section 2

The following question and its specifics should be answered and discussed. Give instances.

Conclusion paragraph

Summarize the essay's main points and draw a conclusion before wrapping up the entire piece.

  • Before you begin writing your response, give the question some thought and make a strategy for it.
  • conform to the word count (no less than 250 words).
  • Write in a respectful tone.
  • Use a content structure with an introduction paragraph at the beginning and a conclusion paragraph at the end, with a main point in between.
  • Answer the question, giving your response and justifications in each body paragraph.

Be a Fluent English Speaker

For your office meetings, presentations, job interviews or ielts.

direct questions essay

Live 1:1 practice, GD, Extempore & Mock Interviews under expert guidance, trusted by over 100,000+ users worldwide, loved by IIT & IAM alumni

direct questions essay

SpeakoClub Features

  • Business English
  • Pronounciation

About SpeakoClub

  • Join SpeakoClub
  • Testimonials
  • Privacy notice
  • Terms of use

Connect SpeakoClub

  • Contact SpeakoClub
  • Follow SpeakoClub on Twitter
  • Follow SpeakoClub on Facebook
  • Subscribe SpeakoClub on YouTube
  • Follow SpeakoClub on Instagram

direct questions essay

Recent posts

  • IELTS Speaking Part 2 Topics: Gift
  • IELTS Speaking Part 2 Topics: Birthday
  • IELTS Speaking Part 2 Topics: Family
  • IELTS Speaking Part 2 Topics: Movie
  • IELTS Speaking Part 2 Topics: Book
  • Link copied!

5 Types of IELTS Essays with Questions and Samples

In IELTS Writing Task 2 (both General and Academic), there are 5 different types of essays:

  • Discussion essay (Discuss both views essay)
  • Agree/disagree essay (Opinion essay)
  • Advantage/disadvantage essay
  • Problem/solution essay
  • Two-part question essay (Direct question essay)

papers on table

Note that no matter what type of IELTS essay you have to write, you need to make sure that you always follow the instructions and write at least 250 words.

IELTS Discussion Essay Questions

IELTS discussion essay asks you to "Discuss both views/sides and give your opinion".

For example:

Some people think that wild animals should not be kept in zoos. Others believe that there are good reasons for having zoos.

Discuss both these views and give your opinion.

See also: Discussion Essay Sample

IELTS Agree/Disagree Essay Questions

IELTS agree/disagree essay, also known as an opinion essay, asks you "Do what extent do you agree?", "Do you agree or disagree?", "What is your opinion?".

Medical procedures for cosmetic purposes should not be allowed.

Do you agree or disagree?

See also: Agree/Disagree Essay Sample

IELTS Advantage/Disadvantage Essay Questions

IELTS advantage/disadvantage essay asks you "What are the advantages of this?", "Do the advantages outweigh the disadvantages?".

Today more and more tourists are visiting places where conditions are difficult, such as the Sahara desert or the Antarctic.

What are the benefits and disadvantages for tourists who visit such places?

See also: Advantage/Disadvantage Essay Sample

IELTS Problem/Solution Essay Questions

IELTS problem/solution essay asks you "What can be done about this problem?", "How could this situation be improved?".

Even though doctors all over the world agree that fast food is bad for people's health, more and more people are eating it.

Why are more people eating fast food?

What can be done about this problem?

See also: Problem/Solution Essay Sample

IELTS Two-part Question Essay Questions

IELTS two-part question essay, also known as direct question essay, asks you to write in response to two or more direct questions.

Millions of people every year move to English speaking countries such as Australia, Britain or America, in order to study at school, college or university.

Why do so many people want to study English?

Why is English such an important international language?

See also: Two-part Question Essay Sample

See more useful IELTS essay resources:

  • How to write an IELTS essay?
  • How to Improve IELTS Writing Task 2 Essay?
  • Essay vocabulary
  • Linking & cohesive words

We are here to help

Whether you have any questions, want to leave feedback or discuss cooperation possibilities, do not hesitate to contact us. We are here to help and will answer as soon as possible. In the meantime, discover our site and let it help you smooth your IELTS journey and make your studies more efficient.

You will find useful information on all the four IELTS test components:

Got a question? We'd love to hear from you!

Preparation for the IELTS Exam

IELTS direct question essay

Ielts two part question essay on homes, model answer for a two-part question essay on homes and housing..

This task was in the Cambridge IELTS 16 book about learning the background of ones home.

At first, you must look at the general topic, which in this case is about housing, then analyse the task to find out what you need to write about and the type of essay it is. In this case, it has two questions in the instruction words so this is a two-part question essay (direct questions essay). Take a look at how I have structured this and given specific examples to support the main ideas.

IELTS writing: model answer on accommodation.

Ielts two-part question essay on renting or owning a home..

Updated : November 9th 2022

A two-part question essay is pretty straightforward. Simply answer the first question in main body one, explain and give an example and then answer the second question in main body two. In the thesis statement briefly answer both questions. See the structure and the model answer below. The task is from Cambridge IELTS 15.

IELTS Mentor "IELTS Preparation & Sample Answer"

  • Skip to content
  • Jump to main navigation and login

Nav view search

  • IELTS Sample

Direct Questions Essay

07 July 2023
07 July 2023
08 July 2023
08 July 2023
10 July 2023
14 July 2023
14 July 2023
15 July 2023
16 July 2023
16 July 2023
17 July 2023
18 July 2023
22 July 2023
26 July 2023
29 July 2023
29 July 2023
01 August 2023
02 August 2023
02 August 2023
04 August 2023

Page 1 of 2

IELTS Materials

  • IELTS Bar Graph
  • IELTS Line Graph
  • IELTS Table Chart
  • IELTS Flow Chart
  • IELTS Pie Chart
  • IELTS Letter Writing
  • IELTS Essay
  • Academic Reading

Useful Links

  • IELTS Secrets
  • Band Score Calculator
  • Exam Specific Tips
  • Useful Websites
  • IELTS Preparation Tips
  • Academic Reading Tips
  • Academic Writing Tips
  • GT Writing Tips
  • Listening Tips
  • Speaking Tips
  • IELTS Grammar Review
  • IELTS Vocabulary
  • IELTS Cue Cards
  • IELTS Life Skills
  • Letter Types

IELTS Mentor - Follow Twitter

  • Privacy Policy
  • Cookie Policy
  • Copyright Notice
  • HTML Sitemap

direct questions essay

45,000+ students realised their study abroad dream with us. Take the first step today

Meet top uk universities from the comfort of your home, here’s your new year gift, one app for all your, study abroad needs, start your journey, track your progress, grow with the community and so much more.

direct questions essay

Verification Code

An OTP has been sent to your registered mobile no. Please verify

direct questions essay

Thanks for your comment !

Our team will review it before it's shown to our readers.

Leverage Edu

  • Management Exams /

ATMA Previous Year Questions: Past Year Papers (Direct Links)

' src=

  • Updated on  
  • Jul 2, 2024

ATMA Previous Year Questions

ATMA exam is conducted for aspirants willing to pursue MBA, PGDM, MCA, and PGDBA programs. The exam is held 4-5 times a year and around 600+ colleges accept the ATMA scores. To ace the exam, it’s imperative that candidates practise the previous year’s papers and mock tests. The blog post below has direct links for ATMA previous year questions papers and also the official mock test link. Continue reading below to find other details.

direct questions essay

Exam NameATMA
Full FormAIMS Test for Management Admissions (ATMA)
Conducting AuthorityAssociation of Indian Management Schools (AIMS)
Exam Duration3 hours
Exam MediumEnglish
Exam ModeCentre-Based Online Mode
Exam Frequency4-5 times a year
Number of Test Centres102 test centres across the country
Exam LevelNational Level
Courses OfferedMBA, PGDM, MCA, PGDBA
Official Websitewww.atmaaims.com

Table of Contents

  • 1 ATMA Exam Latest Update 2024
  • 2 ATMA Previous Year Questions 2024
  • 3 How to Download ATMA Exam Previous Year Question Papers 2024?
  • 4 Tips to Solve ATMA Previous Year Questions 2024

ATMA Exam Latest Update 2024

The ATMA exam June session is all set to be conducted on 23 June 2024. Candidates can download their ATMA admit card from the official website using their PID and password. All the latest update is announced by the exam conducting committee, Association of Indian Management Schools (AIMS). 

ATMA Exam FAQs 2024: Commonly Asked Questions About ATMA Exam 

ATMA Previous Year Questions 2024

The previous year’s question papers are released officially by the exam committee. However, candidates can find the direct link for ATMA previous year sample questions given below: 

Sample Question Paper 12 with Answer: 14 Feb 2021
Sample Question Paper 13 with Answer: 14 Feb 2021
Sample Question Paper 14 with Answer: 14 Feb 2021
Sample Question Paper 15 with Answer: 26 July 2021

Also Read: ATMA Exam 2024 Registration Dates Out Now (June session)

How to Download ATMA Exam Previous Year Question Papers 2024?

Candidates planning to appear for the ATMA exam 2024 must practise the previous year’s papers to understand the question format, exam pattern etc. The easy steps to download the  ATMA Exam Previous Year Question Papers are as follows: 

  • Visit the official website of, the Association of Indian Management Schools (AIMS). 
  • Scroll below and find the “For Applicants” available option. 
  • Several options will appear on the screen. 
  • Select the last option available, “Previous Question Papers with Answers.”
  • The direct links for several ATMA question papers will appear. 
  • Click on the one you want to refer to for practice. 

Also Read: ATMA Exam Dates 2024: Exam Timings (Out Now), Exam Dates, Admit Card (June Phase Out)

Tips to Solve ATMA Previous Year Questions 2024

Solving the ATMA previous year questions will help the candidates to understand the exam pattern, overall and section-wise exam duration, type of questions etc. Other several benefits of solving ATMA previous year questions are listed below: 

  • The previous year paper’s helps candidates to analyse their overall performance in each section. 
  • The sample questions improve the problem-solving ability of candidates. 
  • The regular practice of the previous year’s questions improves the weak areas which candidates need to dedicate their time. 
  •  Candidates will have an idea of how much time to dedicate to the given sections of the ATMA exam 2024. 

Related Reads

Candidates can find their ATMA exam previous papers on the official website of AIMS, ie, https://www.atmaaims.com/index.php.

The exam-conducting committee is the Association of Indian Management Schools (AIMS). 

The ATMA exam is conducted 4 to 5 times a year. 

This was all about the “ATMA Previous Year Questions 2024” . Check out our Management Exams Section , or you can learn more about us by visiting our Indian Universities page.

' src=

Vaishnavi Shukla

Vaishnavi has 2+ years of experience in SEO and Content Marketing. She is highly proficient in English, possessing exceptional language skills and a deep understanding of English grammar and communication. Currently working on Ed Tech, Finance, Lifestyle, and other niches. All her works are infused with love for writing!

Leave a Reply Cancel reply

Save my name, email, and website in this browser for the next time I comment.

Contact no. *

direct questions essay

Connect With Us

45,000+ students realised their study abroad dream with us. take the first step today..

direct questions essay

Resend OTP in

direct questions essay

Need help with?

Study abroad.

UK, Canada, US & More

IELTS, GRE, GMAT & More

Scholarship, Loans & Forex

Country Preference

New Zealand

Which English test are you planning to take?

Which academic test are you planning to take.

Not Sure yet

When are you planning to take the exam?

Already booked my exam slot

Within 2 Months

Want to learn about the test

Which Degree do you wish to pursue?

When do you want to start studying abroad.

January 2024

September 2024

What is your budget to study abroad?

direct questions essay

How would you describe this article ?

Please rate this article

We would like to hear more.

Have something on your mind?

direct questions essay

Make your study abroad dream a reality in January 2022 with

direct questions essay

India's Biggest Virtual University Fair

direct questions essay

Essex Direct Admission Day

Why attend .

direct questions essay

Don't Miss Out

  • Real Estate
  • Public Notices

Facebook Messenger Icon

  • >>&text='+encodeURIComponent(" >>")+'&via=PittsburghPG','','width=500,height=500') "> --> -->
  • Subscribe --> Subscribe
  • Subscriber Services
  • This Just In
  • Event Guide
  • Crimes & Courts
  • Election 2024
  • Health & Wellness
  • Transportation
  • Weather News
  • News Obituaries
  • Environment
  • Faith & Religion
  • Social Services
  • Westmoreland
  • Classifieds
  • Sports Home
  • Sports Columns
  • Gene Collier
  • Jason Mackey
  • Joe Starkey
  • North Shore Drive Podcast
  • Riverhounds
  • Sports Betting
  • Fantasy Football
  • College Sports
  • High School Sports
  • Opinion Home
  • PG Columnists
  • Special to the PG
  • Op-Ed Columns
  • A&E Home
  • Celebrities
  • TV & Radio
  • Concert Listings
  • Theatre & Dance
  • Art & Architecture
  • Buying Here
  • Homes & Gardens
  • Random Acts of Kindness
  • Style & Fashion
  • Business Home
  • Building PGH
  • Business Health
  • Powersource
  • Business / Law
  • Other Business
  • Consumer Alerts
  • Business of Pittsburgh
  • Top Workplaces
  • NEWSLETTERS
  • PUBLIC NOTICES
  • REAL ESTATE
  • CLASSIFIEDS
  • ADVERTISING
  • CAREER OPPORTUNITIES
  • Share full article

Advertisement

How to Watch Biden’s Prime-Time Interview

President Biden is giving his first television interview since last week’s debate to George Stephanopoulos of ABC News, a key moment as he tries to rebound from a poor performance.

direct questions essay

By Neil Vigdor

  • July 5, 2024

President Biden on Friday will sit down with George Stephanopoulos of ABC News for his first television interview since his poor debate performance last week sent his campaign into damage control mode and raised concerns about his ability to stay in the race.

Here’s how to watch it:

The first clip from the interview, which is being taped while Mr. Biden campaigns in Wisconsin, will air on “ World News Tonight with David Muir ” at 6:30 p.m. Eastern time on Friday.

The full conversation will then be broadcast during a prime-time special on ABC starting at 8 p.m., both Eastern and Pacific time.

If you miss it tonight, you have another chance on Sunday: The interview will run again in its entirety on “ This Week with George Stephanopoulos .” Check your local station for air times .

You can also watch in the ABC smartphone app, on ABC.com and via connected devices (Roku, Apple TV+ and Amazon Fire TV).

Neil Vigdor covers politics for The Times, focusing on voting rights issues and election disinformation. More about Neil Vigdor

IELTS Preparation with Liz: Free IELTS Tips and Lessons, 2024

' src=

  • Test Information FAQ
  • Band Scores
  • IELTS Candidate Success Tips
  • Computer IELTS: Pros & Cons
  • How to Prepare
  • Useful Links & Resources
  • Recommended Books
  • Writing Task 1
  • Writing Task 2
  • Speaking Part 1 Topics
  • Speaking Part 2 Topics
  • Speaking Part 3 Topics
  • 100 Essay Questions
  • On The Day Tips
  • Top Results
  • Advanced IELTS

IELTS Model Essay -Two Questions Essay Type

The IELTS Writing Task 2 Two Questions Essay: Causes & Positive/Negative Trends .

There are a number of different types of IELTS essay questions. There are Opinion Essays, Discussion Essays, Advantage/Disadvantage Essays, Solution (including Cause/ Solution) Essays and there are Direct Question Essays (such as the Two Question Essay). However, please note that different teachers use different names for essay types.

The model essay below looks at the Direct Question Essay which contains Two Questions. 

IELTS Essay Question  – Computer Games

More and more adults are playing computer games. Why is this happening? Is it a positive or negative trend?

The main topic in the essay question is Technology and the specific topic is Computer Games. It is a current essay question because it is about a current trend in the world today. I’ve provided a list of tips to help you tackle this type of essay question.

Points to Consider

  • There is only one issue to tackle : computer games. This is lucky. It is an easy essay question. Some essay questions are more complex and have two separate issues to tackle.
  • There are two questions to answer . I call this type of question a “Direct Question Essay”. The first question is about causes of the trend. The second question is about evaluating whether it is good or bad. Whenever you are asked to choose, it means you must give your opinion.
  • Pay attention to the wording of the essay topic . This is about adults, not children. It is about computer games which some people consider are for children. Always pay attention to all keywords in the essay question when you brainstorm or you will go off topic. Ask yourself questions to stimulate ideas. Why are adults playing games on a computer? We know that children like to do this, but why are adults doing this? And is this good? Is it good that adults are playing computer games? If it is bad, why? 
  • Next think about the concept of “computer games”. Spend time analysing the issue given . We often consider computer games to be silly entertainment fore children. But is that correct? Are all computer games actually silly? If we think about this carefully, we will realise that actually some computer games are complex and strategic. Some games require skill and intelligence to play. This means that the issue of computer games is not a simple one. Computer games are varied. Does this essay question apply to childish computer games or complex games? The answer is – it applies to both. So, now we know we can tackle this issue at a level of more depth. Getting to the depth of the issue is essential for a high score.  So, while we can see there is only one issue (computer games) that single issue is complex and can be divided into different aspects.
  • After you brainstorm, choose the ideas that are the most relevant and the easiest to explain well. You don’t get a high score because you have lots of ideas. You get a higher score for presenting specific ideas which are well developed and highly relevant.
  • If you have two questions to cover. It is logical to have two body paragraphs . Being logical in your choice of paragraphing is important. 
  • Provide a clear position in your introduction as to whether you think this is a positive or negative point. Being clear in the introduction helps the examiner follow your body paragraphs more easily and this will increase your score. If you think it is positive – make it clear. If you think it is negative – say so. If you think “it depends” – make sure you word it clearly and explain it clearly in the body paragraphs. The easiest approach is a positive or negative one (a one-sided approach). The “it depends” approach is harder and requires stronger language skills. 

Model Essay: Computer Games (2024)

It seems that the current trend is for an increasing number of adults to enjoy playing computer games in their free time. With the development of game technology, it is hardly surprising that adults are playing games, but whether it is positive or negative depends on the games played and the time spent on them.

In terms of why so many adults are choosing to spend time playing computer games, it is mainly because the technology behind the games is becoming more sophisticated. Initially, when games first came out, they were very simplistic and appealed mainly to children. However, things have moved on since then and games have become visually appealing, very absorbing, require great dexterity and some also have a strategic challenge to them which adults particularly like. Such games can attract professional adults looking to hone tactics and skills to other adults wishing just to relax and switch off.

However, whether this trend in adults towards computer games is beneficial or not can be challenged. Some adults use complex, challenging games as a form of escapism which keeps their mind sharp and helps them relax at the same time. As long as the time spent on such games is balanced with other healthier pursuits, it can be constructive. Unfortunately, adults who ignore their physical health and spend too much time on mindless, repetitive games develop a sedentary lifestyle which can be detrimental to their wellbeing. 

In conclusion, computer games have become more fascinating and tempting to adults. While games that help develop tactics and knowledge might be advantageous, no game, particularly senseless games, should be played to excess and certainly should not replace healthier leisure activities.

Word count = 276 

IELTS Writing Task 2  Model Essays & Tips

Click here to see all model essays, tips etc for writing task 2: IELTS Writing Task 2 Main Page

……………………………………………………

FREE SUBSCRIBE to Get New Lessons & Tips by Email

Type your email…

' src=

I was impressed by your work. It’s really beneficial! Thanks.

' src=

I’m glad it’s helpful 🙂

' src=

This is awesome, there are so much new vocabulary that I can learn from. Thank you Liz! And I wanna ask if all model essays are in the “model essay” category? I’m a new comer and I’m looking for as many well-written writings as possible, like yours!

There are a lot of model essays online. I do not know their quality or how safe they are to use a models. Not all websites are written by professional, experienced teachers who have completed the IELTS examiner training. The models on my essay are safe to use as a guide.

' src=

I received my IELTS result today and I scored 7.5 overall band score. I can’t thank you enough for your valuable help and guidance.

More than IELTS, I’m more confident than ever and look forward to continuing this learning further.

Thanks again and take care.

Best Regards, Kamlesh

I’m so pleased for you, Kamlesh! Very well done to you!! I do hope you continue learning. One day I plan to start an English Liz Youtube channel so that people can keep learning beyond their IELTS test 🙂

' src=

That would be perfect <3

' src=

Thank you so much, Liz. I really appreciate your fantastic work.

You’re welcome 🙂

' src=

Good evening Liz. I’m grateful for your guidance and tutelage as I scored 7.5 in my writing, 7.5 in speaking, 7.0 in listening and 6.0 in reading after just a short time with you. I’m optimistic in my next attempt I should get the desired scores. You’re a great teacher ma’am.

I wish you lots of luck in your next test 🙂

' src=

Rituparna Saha says April 5, 2024 Thank you Liz for all your support and guidance on Writing Task 2. I greatly appreciate your efforts.

' src=

I took my IELTS test a few days ago. The results are out and I got a band 7 in writing with an overall band 7.5 in just a week. For writing I only watched your videos and took notes of all the points you taught. I did not even practice writing much, just referred to your videos and read all the materials on the website. Your content is pure gold and you are an amazing teacher. Ilysm

Very well done with your results!! Many people struggle to hit band 7 in writing. Congrats!

' src=

this is Soo nice Liz I have been following you and your materials are helpful kindly would like to know where I can download the Cambridge book or if you can share any regards Hellen

The IELTS Cambridge Test Books are copyrighted so I can’t share them. However, you can find new as well as second hand copies on Amazon or possibly in a local educational store.

' src=

I don’t feel the introduction is clear, and there is no clear opinion.

“it depends” indicates the opinion. It shows that the writer intends to be specific about when it is positive and when it is negative because their opinion covers both. This can often be the case with IELTS essays that require an opinion. You do not have to choose positive or negative and be 100% on one side. It is 100% acceptable in IELTS but it is a more difficult opinion to create if one’s English language isn’t strong.

' src=

Hello Liz, is it okay to write a contrast (one point) before the conclusion paragraph in agree or disagree essay? Thank you.

This is not an agree/disagree essay. The Opinion Essay is an agree disagree essay which requires you to agree, disagree or have a partial agreement with an opinion given by IELTS. That essay is not an Opinion Essay because you aren’t being asked to respond to an opinion given by IELTS. This essay is a Direct Questions Essay which may or may not require an opinion depending on the questions you are given. In an Opinion Essay, you introduce your opinion in the introduction and the whole essay explains your opinion. You can’t suddenly put a different opinion further down the essay. Your opinion must be consistent throughout the whole essay. I recommend you get my advanced lessons because they explain in detail how to tackle an Opinion Essay. You can find them in my online store: https://elizabethferguson.podia.com/

' src=

Thanks a lot liz.

' src=

Hi dear Liz. Hope you are fast recuperating. I have written the following intro. ” There is a growing propensity among the youth to play computer games. This is due to indulgence of parents and can have possible detrimental effects.” I know you don’t comment on write ups, but this is with a hope, in case…

I’ll just make one comment. I made a list of points to consider. Point 3 was important. This isn’t about youths. It’s about adults, which means people in their early 20’s, 30’s, 40’s, 50’s etc. And because it is about adults, it cannot be related to “indulgence from parents”. If you make this mistake, most of your essay will be off topic. That is the reason I wrote point 3. Take a look again because it’s an important lesson to learn.

' src=

This writing test sample answer makes a whole lots of sense to me. Well appreciated 👍👍👍.

I’m glad it made sense. IELTS isn’t difficult once you understand more about the test and the aims you should have.

' src=

Hi, Due to the financial crisis, I lacked many things like IELTS practice/preparation classes and missed many classes from a good teacher like you.

therefore, if you have any better offer like a full free studentship & give me the opportunity. Thank you in advance for your kind coope

This website has hundreds of page of free practice lessons, tips, topics, videos, advice, information, model answers etc. Use them well. Learn from each page and take your time. Then use the IELTS Cambridge test books for full test practice at home.

' src=

Thank you so much Liz for your valuable tips and techniques 🙏❤️

I’m glad it was helpful 🙂

' src=

Thank you for this it’s very helpful Liz. I greatly appreciate your efforts

Speak Your Mind Cancel reply

Notify me of follow-up comments by email.

Notify me of new posts by email.

Advanced IELTS Lessons & E-books

direct questions essay

Recent Lessons

Ielts writing task 2 essay topics 2024, introduction paragraph for ielts writing task 1, ielts speaking part 2 topic water sports: vocab & model answer, ielts liz personal update 2024, ielts bar chart of age groups 2024.

' src=

Click Below to Learn:

  • IELTS Test Information

Copyright Notice

Copyright © Elizabeth Ferguson, 2014 – 2024

All rights reserved.

Privacy Policy & Disclaimer

  • Click here:  Privacy Policy 
  • Click here: Disclaimer

Return to top of page

Copyright © 2024 · Prose on Genesis Framework · WordPress · Log in

Human Subjects Office

Medical terms in lay language.

Please use these descriptions in place of medical jargon in consent documents, recruitment materials and other study documents. Note: These terms are not the only acceptable plain language alternatives for these vocabulary words.

This glossary of terms is derived from a list copyrighted by the University of Kentucky, Office of Research Integrity (1990).

For clinical research-specific definitions, see also the Clinical Research Glossary developed by the Multi-Regional Clinical Trials (MRCT) Center of Brigham and Women’s Hospital and Harvard  and the Clinical Data Interchange Standards Consortium (CDISC) .

Alternative Lay Language for Medical Terms for use in Informed Consent Documents

A   B   C   D   E   F   G   H   I  J  K   L   M   N   O   P   Q   R   S   T   U   V   W  X  Y  Z

ABDOMEN/ABDOMINAL body cavity below diaphragm that contains stomach, intestines, liver and other organs ABSORB take up fluids, take in ACIDOSIS condition when blood contains more acid than normal ACUITY clearness, keenness, esp. of vision and airways ACUTE new, recent, sudden, urgent ADENOPATHY swollen lymph nodes (glands) ADJUVANT helpful, assisting, aiding, supportive ADJUVANT TREATMENT added treatment (usually to a standard treatment) ANTIBIOTIC drug that kills bacteria and other germs ANTIMICROBIAL drug that kills bacteria and other germs ANTIRETROVIRAL drug that works against the growth of certain viruses ADVERSE EFFECT side effect, bad reaction, unwanted response ALLERGIC REACTION rash, hives, swelling, trouble breathing AMBULATE/AMBULATION/AMBULATORY walk, able to walk ANAPHYLAXIS serious, potentially life-threatening allergic reaction ANEMIA decreased red blood cells; low red cell blood count ANESTHETIC a drug or agent used to decrease the feeling of pain, or eliminate the feeling of pain by putting you to sleep ANGINA pain resulting from not enough blood flowing to the heart ANGINA PECTORIS pain resulting from not enough blood flowing to the heart ANOREXIA disorder in which person will not eat; lack of appetite ANTECUBITAL related to the inner side of the forearm ANTIBODY protein made in the body in response to foreign substance ANTICONVULSANT drug used to prevent seizures ANTILIPEMIC a drug that lowers fat levels in the blood ANTITUSSIVE a drug used to relieve coughing ARRHYTHMIA abnormal heartbeat; any change from the normal heartbeat ASPIRATION fluid entering the lungs, such as after vomiting ASSAY lab test ASSESS to learn about, measure, evaluate, look at ASTHMA lung disease associated with tightening of air passages, making breathing difficult ASYMPTOMATIC without symptoms AXILLA armpit

BENIGN not malignant, without serious consequences BID twice a day BINDING/BOUND carried by, to make stick together, transported BIOAVAILABILITY the extent to which a drug or other substance becomes available to the body BLOOD PROFILE series of blood tests BOLUS a large amount given all at once BONE MASS the amount of calcium and other minerals in a given amount of bone BRADYARRHYTHMIAS slow, irregular heartbeats BRADYCARDIA slow heartbeat BRONCHOSPASM breathing distress caused by narrowing of the airways

CARCINOGENIC cancer-causing CARCINOMA type of cancer CARDIAC related to the heart CARDIOVERSION return to normal heartbeat by electric shock CATHETER a tube for withdrawing or giving fluids CATHETER a tube placed near the spinal cord and used for anesthesia (indwelling epidural) during surgery CENTRAL NERVOUS SYSTEM (CNS) brain and spinal cord CEREBRAL TRAUMA damage to the brain CESSATION stopping CHD coronary heart disease CHEMOTHERAPY treatment of disease, usually cancer, by chemical agents CHRONIC continuing for a long time, ongoing CLINICAL pertaining to medical care CLINICAL TRIAL an experiment involving human subjects COMA unconscious state COMPLETE RESPONSE total disappearance of disease CONGENITAL present before birth CONJUNCTIVITIS redness and irritation of the thin membrane that covers the eye CONSOLIDATION PHASE treatment phase intended to make a remission permanent (follows induction phase) CONTROLLED TRIAL research study in which the experimental treatment or procedure is compared to a standard (control) treatment or procedure COOPERATIVE GROUP association of multiple institutions to perform clinical trials CORONARY related to the blood vessels that supply the heart, or to the heart itself CT SCAN (CAT) computerized series of x-rays (computerized tomography) CULTURE test for infection, or for organisms that could cause infection CUMULATIVE added together from the beginning CUTANEOUS relating to the skin CVA stroke (cerebrovascular accident)

DERMATOLOGIC pertaining to the skin DIASTOLIC lower number in a blood pressure reading DISTAL toward the end, away from the center of the body DIURETIC "water pill" or drug that causes increase in urination DOPPLER device using sound waves to diagnose or test DOUBLE BLIND study in which neither investigators nor subjects know what drug or treatment the subject is receiving DYSFUNCTION state of improper function DYSPLASIA abnormal cells

ECHOCARDIOGRAM sound wave test of the heart EDEMA excess fluid collecting in tissue EEG electric brain wave tracing (electroencephalogram) EFFICACY effectiveness ELECTROCARDIOGRAM electrical tracing of the heartbeat (ECG or EKG) ELECTROLYTE IMBALANCE an imbalance of minerals in the blood EMESIS vomiting EMPIRIC based on experience ENDOSCOPIC EXAMINATION viewing an  internal part of the body with a lighted tube  ENTERAL by way of the intestines EPIDURAL outside the spinal cord ERADICATE get rid of (such as disease) Page 2 of 7 EVALUATED, ASSESSED examined for a medical condition EXPEDITED REVIEW rapid review of a protocol by the IRB Chair without full committee approval, permitted with certain low-risk research studies EXTERNAL outside the body EXTRAVASATE to leak outside of a planned area, such as out of a blood vessel

FDA U.S. Food and Drug Administration, the branch of federal government that approves new drugs FIBROUS having many fibers, such as scar tissue FIBRILLATION irregular beat of the heart or other muscle

GENERAL ANESTHESIA pain prevention by giving drugs to cause loss of consciousness, as during surgery GESTATIONAL pertaining to pregnancy

HEMATOCRIT amount of red blood cells in the blood HEMATOMA a bruise, a black and blue mark HEMODYNAMIC MEASURING blood flow HEMOLYSIS breakdown in red blood cells HEPARIN LOCK needle placed in the arm with blood thinner to keep the blood from clotting HEPATOMA cancer or tumor of the liver HERITABLE DISEASE can be transmitted to one’s offspring, resulting in damage to future children HISTOPATHOLOGIC pertaining to the disease status of body tissues or cells HOLTER MONITOR a portable machine for recording heart beats HYPERCALCEMIA high blood calcium level HYPERKALEMIA high blood potassium level HYPERNATREMIA high blood sodium level HYPERTENSION high blood pressure HYPOCALCEMIA low blood calcium level HYPOKALEMIA low blood potassium level HYPONATREMIA low blood sodium level HYPOTENSION low blood pressure HYPOXEMIA a decrease of oxygen in the blood HYPOXIA a decrease of oxygen reaching body tissues HYSTERECTOMY surgical removal of the uterus, ovaries (female sex glands), or both uterus and ovaries

IATROGENIC caused by a physician or by treatment IDE investigational device exemption, the license to test an unapproved new medical device IDIOPATHIC of unknown cause IMMUNITY defense against, protection from IMMUNOGLOBIN a protein that makes antibodies IMMUNOSUPPRESSIVE drug which works against the body's immune (protective) response, often used in transplantation and diseases caused by immune system malfunction IMMUNOTHERAPY giving of drugs to help the body's immune (protective) system; usually used to destroy cancer cells IMPAIRED FUNCTION abnormal function IMPLANTED placed in the body IND investigational new drug, the license to test an unapproved new drug INDUCTION PHASE beginning phase or stage of a treatment INDURATION hardening INDWELLING remaining in a given location, such as a catheter INFARCT death of tissue due to lack of blood supply INFECTIOUS DISEASE transmitted from one person to the next INFLAMMATION swelling that is generally painful, red, and warm INFUSION slow injection of a substance into the body, usually into the blood by means of a catheter INGESTION eating; taking by mouth INTERFERON drug which acts against viruses; antiviral agent INTERMITTENT occurring (regularly or irregularly) between two time points; repeatedly stopping, then starting again INTERNAL within the body INTERIOR inside of the body INTRAMUSCULAR into the muscle; within the muscle INTRAPERITONEAL into the abdominal cavity INTRATHECAL into the spinal fluid INTRAVENOUS (IV) through the vein INTRAVESICAL in the bladder INTUBATE the placement of a tube into the airway INVASIVE PROCEDURE puncturing, opening, or cutting the skin INVESTIGATIONAL NEW DRUG (IND) a new drug that has not been approved by the FDA INVESTIGATIONAL METHOD a treatment method which has not been proven to be beneficial or has not been accepted as standard care ISCHEMIA decreased oxygen in a tissue (usually because of decreased blood flow)

LAPAROTOMY surgical procedure in which an incision is made in the abdominal wall to enable a doctor to look at the organs inside LESION wound or injury; a diseased patch of skin LETHARGY sleepiness, tiredness LEUKOPENIA low white blood cell count LIPID fat LIPID CONTENT fat content in the blood LIPID PROFILE (PANEL) fat and cholesterol levels in the blood LOCAL ANESTHESIA creation of insensitivity to pain in a small, local area of the body, usually by injection of numbing drugs LOCALIZED restricted to one area, limited to one area LUMEN the cavity of an organ or tube (e.g., blood vessel) LYMPHANGIOGRAPHY an x-ray of the lymph nodes or tissues after injecting dye into lymph vessels (e.g., in feet) LYMPHOCYTE a type of white blood cell important in immunity (protection) against infection LYMPHOMA a cancer of the lymph nodes (or tissues)

MALAISE a vague feeling of bodily discomfort, feeling badly MALFUNCTION condition in which something is not functioning properly MALIGNANCY cancer or other progressively enlarging and spreading tumor, usually fatal if not successfully treated MEDULLABLASTOMA a type of brain tumor MEGALOBLASTOSIS change in red blood cells METABOLIZE process of breaking down substances in the cells to obtain energy METASTASIS spread of cancer cells from one part of the body to another METRONIDAZOLE drug used to treat infections caused by parasites (invading organisms that take up living in the body) or other causes of anaerobic infection (not requiring oxygen to survive) MI myocardial infarction, heart attack MINIMAL slight MINIMIZE reduce as much as possible Page 4 of 7 MONITOR check on; keep track of; watch carefully MOBILITY ease of movement MORBIDITY undesired result or complication MORTALITY death MOTILITY the ability to move MRI magnetic resonance imaging, diagnostic pictures of the inside of the body, created using magnetic rather than x-ray energy MUCOSA, MUCOUS MEMBRANE moist lining of digestive, respiratory, reproductive, and urinary tracts MYALGIA muscle aches MYOCARDIAL pertaining to the heart muscle MYOCARDIAL INFARCTION heart attack

NASOGASTRIC TUBE placed in the nose, reaching to the stomach NCI the National Cancer Institute NECROSIS death of tissue NEOPLASIA/NEOPLASM tumor, may be benign or malignant NEUROBLASTOMA a cancer of nerve tissue NEUROLOGICAL pertaining to the nervous system NEUTROPENIA decrease in the main part of the white blood cells NIH the National Institutes of Health NONINVASIVE not breaking, cutting, or entering the skin NOSOCOMIAL acquired in the hospital

OCCLUSION closing; blockage; obstruction ONCOLOGY the study of tumors or cancer OPHTHALMIC pertaining to the eye OPTIMAL best, most favorable or desirable ORAL ADMINISTRATION by mouth ORTHOPEDIC pertaining to the bones OSTEOPETROSIS rare bone disorder characterized by dense bone OSTEOPOROSIS softening of the bones OVARIES female sex glands

PARENTERAL given by injection PATENCY condition of being open PATHOGENESIS development of a disease or unhealthy condition PERCUTANEOUS through the skin PERIPHERAL not central PER OS (PO) by mouth PHARMACOKINETICS the study of the way the body absorbs, distributes, and gets rid of a drug PHASE I first phase of study of a new drug in humans to determine action, safety, and proper dosing PHASE II second phase of study of a new drug in humans, intended to gather information about safety and effectiveness of the drug for certain uses PHASE III large-scale studies to confirm and expand information on safety and effectiveness of new drug for certain uses, and to study common side effects PHASE IV studies done after the drug is approved by the FDA, especially to compare it to standard care or to try it for new uses PHLEBITIS irritation or inflammation of the vein PLACEBO an inactive substance; a pill/liquid that contains no medicine PLACEBO EFFECT improvement seen with giving subjects a placebo, though it contains no active drug/treatment PLATELETS small particles in the blood that help with clotting POTENTIAL possible POTENTIATE increase or multiply the effect of a drug or toxin (poison) by giving another drug or toxin at the same time (sometimes an unintentional result) POTENTIATOR an agent that helps another agent work better PRENATAL before birth PROPHYLAXIS a drug given to prevent disease or infection PER OS (PO) by mouth PRN as needed PROGNOSIS outlook, probable outcomes PRONE lying on the stomach PROSPECTIVE STUDY following patients forward in time PROSTHESIS artificial part, most often limbs, such as arms or legs PROTOCOL plan of study PROXIMAL closer to the center of the body, away from the end PULMONARY pertaining to the lungs

QD every day; daily QID four times a day

RADIATION THERAPY x-ray or cobalt treatment RANDOM by chance (like the flip of a coin) RANDOMIZATION chance selection RBC red blood cell RECOMBINANT formation of new combinations of genes RECONSTITUTION putting back together the original parts or elements RECUR happen again REFRACTORY not responding to treatment REGENERATION re-growth of a structure or of lost tissue REGIMEN pattern of giving treatment RELAPSE the return of a disease REMISSION disappearance of evidence of cancer or other disease RENAL pertaining to the kidneys REPLICABLE possible to duplicate RESECT remove or cut out surgically RETROSPECTIVE STUDY looking back over past experience

SARCOMA a type of cancer SEDATIVE a drug to calm or make less anxious SEMINOMA a type of testicular cancer (found in the male sex glands) SEQUENTIALLY in a row, in order SOMNOLENCE sleepiness SPIROMETER an instrument to measure the amount of air taken into and exhaled from the lungs STAGING an evaluation of the extent of the disease STANDARD OF CARE a treatment plan that the majority of the medical community would accept as appropriate STENOSIS narrowing of a duct, tube, or one of the blood vessels in the heart STOMATITIS mouth sores, inflammation of the mouth STRATIFY arrange in groups for analysis of results (e.g., stratify by age, sex, etc.) STUPOR stunned state in which it is difficult to get a response or the attention of the subject SUBCLAVIAN under the collarbone SUBCUTANEOUS under the skin SUPINE lying on the back SUPPORTIVE CARE general medical care aimed at symptoms, not intended to improve or cure underlying disease SYMPTOMATIC having symptoms SYNDROME a condition characterized by a set of symptoms SYSTOLIC top number in blood pressure; pressure during active contraction of the heart

TERATOGENIC capable of causing malformations in a fetus (developing baby still inside the mother’s body) TESTES/TESTICLES male sex glands THROMBOSIS clotting THROMBUS blood clot TID three times a day TITRATION a method for deciding on the strength of a drug or solution; gradually increasing the dose T-LYMPHOCYTES type of white blood cells TOPICAL on the surface TOPICAL ANESTHETIC applied to a certain area of the skin and reducing pain only in the area to which applied TOXICITY side effects or undesirable effects of a drug or treatment TRANSDERMAL through the skin TRANSIENTLY temporarily TRAUMA injury; wound TREADMILL walking machine used to test heart function

UPTAKE absorbing and taking in of a substance by living tissue

VALVULOPLASTY plastic repair of a valve, especially a heart valve VARICES enlarged veins VASOSPASM narrowing of the blood vessels VECTOR a carrier that can transmit disease-causing microorganisms (germs and viruses) VENIPUNCTURE needle stick, blood draw, entering the skin with a needle VERTICAL TRANSMISSION spread of disease

WBC white blood cell

IMAGES

  1. How To Write A Direct Question Essay

    direct questions essay

  2. IELTS Direct Question Essay on Society With Sample Answer

    direct questions essay

  3. How To Write A Direct Question Essay

    direct questions essay

  4. How To Write A Direct Question Essay

    direct questions essay

  5. IELTS Direct Question Essay Sample 1

    direct questions essay

  6. IELTS Writing Task 2

    direct questions essay

VIDEO

  1. BECE TRIAL QUESTIONS-ESSAY (PAPER 2)

  2. PLUS TWO BUSINESS STUDIES

  3. Plustwo Economics

  4. Direct speech indirect speech Rule-3 (part-2) || JSJ JESY ENGLISH GRAMMAR

  5. Double part questions essay (2-qism)

  6. Chỉnh Bài Mẫu Essay 6.5

COMMENTS

  1. IELTS Direct Question Essays

    Direct Question Essay: A Skill to Learn! In conclusion, mastering the art of writing Direct Question Essays is achievable with practice, a clear understanding of the format, and attention to key pointers. By following the structure, incorporating the tips provided, and analyzing the sample questions and answers, you can significantly enhance ...

  2. IELTS Direct Questions Sample Essay Titles

    Below is a list of sample essay questions for IELTS direct questions essays which can come in writing task 2. These essay types are also known as two question essays (although it is possible to get more than two questions). Direct Questions Essay Tips. spend time planning the answers to the questions; each question must have one main point as ...

  3. Direct Question Essay IELTS: Structure, Sample & Tips 2024

    This essay type tests your ability to provide clear, coherent, and relevant answers, showcasing your language proficiency and critical thinking skills. Importance of Direct Question Essay in IELTS Writing Task 2. Direct Question Essay in IELTS plays a significant role in IELTS Writing Task 2, often appearing in exams.

  4. IELTS Model Essay Score 9 for Direct Questions

    The direct question essay contains two or more questions to answer. However, both require an opinion. The catgorises are mainly used by teachers in order to teach - so don't worry so much. Just follow the instructions. Reply. Haribol says November 6, 2016 at . Thanks for help. Reply.

  5. IELTS Direct Question Essay: tips, common mistakes, questions & essays

    Step 4: Structure Your Essay. The final step in the planning process is to structure your essay. This simply means deciding which main ideas to put in which paragraphs. I would recommend a simple structure like this: Paragraph 1: introduce the essay. Paragraph 2: discuss your answer to the 1st question.

  6. IELTS Double Question Essays

    It's easy to learn and will enable you to quickly plan and write a high-level essay. 1) Introduction. Paraphrase the question. Outline sentence - state your answer to both questions. 2) Main body paragraph 1 - Answer question 1. Topic sentence - state your answer. Explanation - develop the idea.

  7. Direct Question Essay in IELTS: Everything You Need to Know

    Direct Question Essay IELTS: The IELTS Writing Task 2 demands test takers to write cohesive, nuanced, and insightful essays. This section includes a diverse spectrum of essay types, such as opinion pieces, problem-solution analyses, and advantage/disadvantage examinations. Notably, the Direct Question Essay, characterized by its explicit and directive nature, demands a clear and concise ...

  8. Full IELTS Academic Writing Task 2 Band 9 Tips and Sample Essay| Direct

    In this video recorded by Ross IELTS Academy, you will get familiar with how to deal with a Direct question in the Academic IELTS Writing Task2.Here's a full...

  9. IELTS Writing Task 2: Two-Part Questions

    For example, a problem and solution essay is two parts. However, what we mean by a "two-part question" is one that contains two questions. The reason this is sometimes called a "direct question" task is that the questions themselves are more direct than other IELTS writing task 2 question types, which instruct candidates to explore an idea.

  10. Direct Question Essay IELTS 2024: Tips, Structure, Sample

    Looking for Direct Question Essay IELTS tips Click to learn how to plan, recognise prompts get samples for a successful IELTS essay! 9/26/2023 2 min read. In the world of standardized English language proficiency tests, the IELTS (International English Language Testing System) is a formidable challenge for many. One of its key components, the ...

  11. IELTS Writing Task 2 Essay Structures + Band 9 Essays

    The five most common IELTS Writing Task 2 questions are: Opinion (Agree or Disagree) Advantages and Disadvantages. Problem and Solution. Discussion (Discuss both views) Two-part Question. Below I will outline examples and a structure approved by experienced IELTS teachers and examiners for each type of question.

  12. Direct Question Essays

    In this free interactive webinar, our IELTS expert Annelie will guide us through how to answer a direct question essay in Academic Writing Task 2. This webin...

  13. Direct Question Essay Examples

    IELTS Direct Question Essay Sample 4 - Education IELTS Direct Question Essay Sample 4 - Education. IELTS Writing Task 2 direct question essay example that is a band score 8. The question is: Some people think that children should be homeschooled when they are very young while others think it is better for them to attend a kindergarten ...

  14. IELTS Direct Question Essay Model Answer: Education

    Instructor Feedback on IELTS Direct Question Essay: Education. Task Achievement - The essay provides an answer to the question asked, supported by relevant examples.. Coherence and Cohesion - The answer has been divided into clear logical paragraphs and each main body paragraph only has one main idea.There are cohesive links between the main body paragraphs.

  15. 100 IELTS Essay Questions

    Transport (7 essay questions) Work (17 essay questions) 2) IELTS Essay Questions by Essay Type There are 5 main types of essay questions in IELTS writing task 2 (opinion essays, discussion essay, advantage/disadvantage essays, solution essay and direct question essays). Click on the links below to see some sample essay questions for each type.

  16. Direct Question Essays

    Tips. Before you begin writing your response, give the question some thought and make a strategy for it. conform to the word count (no less than 250 words). Write in a respectful tone. Use a content structure with an introduction paragraph at the beginning and a conclusion paragraph at the end, with a main point in between.

  17. 5 Types of IELTS Essays with Questions and Samples

    IELTS two-part question essay, also known as direct question essay, asks you to write in response to two or more direct questions. For example: Millions of people every year move to English speaking countries such as Australia, Britain or America, in order to study at school, college or university.

  18. My IELTS Classroom

    Most direct question essays are actually one of the more common essays in disguise. Learn how to deal with a genuine direct question in terms of organisation & language. Writing Speaking Reading Listening Pick My Package Writing Speaking Reading Listening ...

  19. IELTS Direct Question Essay on Society With Sample Answer

    Tips to write IELTS Direct question essay: IELTS Direct Question Essay Types are also known as two question essays. Must spend time planning the answers to the questions; Put the answer to each question in a separate body paragraph; Question: In many countries, the tradition of families having meals together is disappearing. Why is this happening?

  20. IELTS direct question essay

    A two-part question essay is pretty straightforward. Simply answer the first question in main body one, explain and give an example and then answer the second question in main body two. In the thesis statement briefly answer both questions. See the structure and the model answer below. The task is from Cambridge IELTS 15.

  21. Direct Questions Essay

    IELTS Essay # 1343 - People may no longer be able to pay for things using cash. 02 August 2023. IELTS Essay # 1344 - Performance of staff can have a significant impact on the success of a company. 02 August 2023. IELTS Essay # 571 - Far too little has been done to prevent animals and plants from dying out.

  22. ATMA Previous Year Questions: Past Year Papers (Direct Links)

    Select the last option available, "Previous Question Papers with Answers." The direct links for several ATMA question papers will appear. Click on the one you want to refer to for practice. Also Read: ATMA Exam Dates 2024: Exam Timings (Out Now), Exam Dates, Admit Card (June Phase Out) Tips to Solve ATMA Previous Year Questions 2024

  23. Mindblindness : An Essay on Autism and Theory of Mind

    In Mindblindness, Simon Baron-Cohen presents a model of the evolution and development of "mindreading."He argues that we mindread all the time, effortlessly, automatically, and mostly unconsciously. It is the natural way in which we interpret, predict, and participate in social behavior and communication.

  24. More colleges, including some in Western Pa., are admitting students

    Write out personal and family history. Share activities and leadership positions. Craft and proofread essays. Pay a fee, if applicable. Repeat. Those are...

  25. How to Watch Biden's ABC News Interview With George Stephanopoulos

    President Biden is giving his first television interview since last week's debate to George Stephanopoulos of ABC News, a key moment as he tries to rebound from a poor performance. By Neil ...

  26. IELTS Model Essay -Two Questions Essay Type

    There are two questions to answer. I call this type of question a "Direct Question Essay". The first question is about causes of the trend. The second question is about evaluating whether it is good or bad. Whenever you are asked to choose, it means you must give your opinion. Pay attention to the wording of the essay topic.

  27. Dr. Sanjay Gupta: It's time for President Biden to undergo detailed

    The White House has rejected requests from the press to release more medical records and question Biden's physician, Dr. Kevin O'Connor. Biden's press secretary said O'Connor watched the ...

  28. What would happen if Biden decided to leave the race?

    Joe Biden's spotty debate performance immediately triggered new questions from worried Democrats about whether he would leave the presidential race.

  29. Direct Question Essays

    In this free interactive webinar, our IELTS expert Stephen will guide us through how to answer a direct question essay in Academic Writing: Task 2. This Live...

  30. Medical Terms in Lay Language

    Human Subjects Office / IRB Hardin Library, Suite 105A 600 Newton Rd Iowa City, IA 52242-1098. Voice: 319-335-6564 Fax: 319-335-7310